Jump to content

Wikipedia:Reference desk/Science: Difference between revisions

From Wikipedia, the free encyclopedia
Content deleted Content added
m ā†’ā€ŽUFO identification: thanks for the gallery
Line 550: Line 550:
I must admit I also have never heard of a historical case of killing in this way, but then again, my character isn't much for history. Nor is he very experienced in the killing business, shall we say. This is the best method he could come up with when presented with a drop from elevation onto the target. [[User:RyuGenkai|RyuGenkai]] ([[User talk:RyuGenkai|talk]]) 16:14, 3 November 2009 (UTC)
I must admit I also have never heard of a historical case of killing in this way, but then again, my character isn't much for history. Nor is he very experienced in the killing business, shall we say. This is the best method he could come up with when presented with a drop from elevation onto the target. [[User:RyuGenkai|RyuGenkai]] ([[User talk:RyuGenkai|talk]]) 16:14, 3 November 2009 (UTC)
:What sort of author capitalizes common nouns like 'physiology,' 'clavicle' and 'scapula.' '''[[User:DRosenbach|<span style="color:#006400">DRosenbach</span>]]''' <sup>([[User_talk:DRosenbach|<span style="color:#006400">Talk</span>]] | [[Special:Contributions/DRosenbach|<span style="color:#006400">Contribs</span>]])</sup> 20:43, 3 November 2009 (UTC)
:What sort of author capitalizes common nouns like 'physiology,' 'clavicle' and 'scapula.' '''[[User:DRosenbach|<span style="color:#006400">DRosenbach</span>]]''' <sup>([[User_talk:DRosenbach|<span style="color:#006400">Talk</span>]] | [[Special:Contributions/DRosenbach|<span style="color:#006400">Contribs</span>]])</sup> 20:43, 3 November 2009 (UTC)
: The film [[Torn Curtain]] is notable for a murder scene "that Hitchcock made specifically to show the audience how difficult it is to kill a man". [[Special:Contributions/81.131.65.113|81.131.65.113]] ([[User talk:81.131.65.113|talk]]) 21:38, 3 November 2009 (UTC)


== melanin benefit vs cancer risk ==
== melanin benefit vs cancer risk ==

Revision as of 21:38, 3 November 2009

Welcome to the science section
of the Wikipedia reference desk.
Select a section:
Want a faster answer?

Main page: Help searching Wikipedia

Ā Ā Ā 

How can I get my question answered?

  • Select the section of the desk that best fits the general topic of your question (see the navigation column to the right).
  • Post your question to only one section, providing a short header that gives the topic of your question.
  • Type '~~~~' (that is, four tilde characters) at the end ā€“ this signs and dates your contribution so we know who wrote what and when.
  • Don't post personal contact information ā€“ it will be removed. Any answers will be provided here.
  • Please be as specific as possible, and include all relevant context ā€“ the usefulness of answers may depend on the context.
  • Note:
    • We don't answer (and may remove) questions that require medical diagnosis or legal advice.
    • We don't answer requests for opinions, predictions or debate.
    • We don't do your homework for you, though we'll help you past the stuck point.
    • We don't conduct original research or provide a free source of ideas, but we'll help you find information you need.



How do I answer a question?

Main page: Wikipedia:Reference desk/Guidelines

  • The best answers address the question directly, and back up facts with wikilinks and links to sources. Do not edit others' comments and do not give any medical or legal advice.
See also:



October 28

Food safety question

So I bought this delicious jar of imported sun-dried tomatoes in olive oil. Yum. Trouble is I don't know where to keep it now that the jar is open. If I put in the fridge, the olive oil congeals into an unappetizing, solid yellow cylinder. If I keep it the cupboard I worry about bacteria breeding in the oil. What's a Fat Man to do?--The Fat Man Who Never Came Back (talk) 00:03, 28 October 2009 (UTC)[reply]

Does it not have storage instructions on the label? --Tango (talk) 00:11, 28 October 2009 (UTC)[reply]
No (it's from Turkey; they don't have food safety over there).--The Fat Man Who Never Came Back (talk) 00:13, 28 October 2009 (UTC)[reply]
What's a fat man to do? Eat them, of course! Seriously, if you don't let them get damp, they should keep for at least a week out of the fridge -- there's basically no form of life that can grow without water. Looie496 (talk) 00:19, 28 October 2009 (UTC)[reply]
Oil and water don't mix very well. Safety isn't a big issue, and you may want to read water activity as an explanation of why bacteria breeding is not a big deal for things packed in oil. Having the oil go rancid from exposure to air is more of a quality concern than a safety issue. If there isn't any water in it, there won't be any bacteria growing either. There are still bacteria there, but most are spore-formers and other inactive forms. SDY (talk) 00:20, 28 October 2009 (UTC)[reply]
The stop them getting damp, you should make sure they are all completely covered by the oil. If they aren't, add more oil. --Tango (talk) 00:21, 28 October 2009 (UTC)[reply]
They were sun dried to remove the water then put in oil to keep them away from bacteria. This method was invented before fridges were even thought of. Put them where you like and they'll last for months. 08:32, 28 October 2009 (UTC) ā€”Preceding unsigned comment added by Richard Avery (talk ā€¢ contribs)
That is a good general point - a lot of foods were invented as ways of preserving fruits and vegetables over winter (jams/jellys, chutneys, pickles, etc.). They will last months without us doing anything further to preserve them. --Tango (talk) 15:49, 28 October 2009 (UTC)[reply]
Thank you, your answers make sense. However, perhaps what provoked my original question was a chef I respect once warning us to be very careful when making homemade garlic-infused or herb-infused oils, as improper storage or handling can result in botulism contamination. Why would botulism bacteria grow, even if the product were submersed in oil? Is this because of the water content of the garlic or herbs?--The Fat Man Who Never Came Back (talk) 00:02, 29 October 2009 (UTC)[reply]
I can't say anything about the safety, but I will note that submersion in fat (Confit) has a long history as a food preservation technique. -- 128.104.112.149 (talk) 22:31, 29 October 2009 (UTC)[reply]

Voice differences between blondes and brunettes

After comparing some samples I have a feeling, that brunette girls might have a higher pitch than blondes, who feature a bit deeper voice as compared to brunettes. Are there actually any differences in general (including red-haired girls)? ā€”Preceding unsigned comment added by 217.25.31.177 (talk) 00:51, 28 October 2009 (UTC)[reply]

Perform a double blind randomized clinical trial and let us know. DRosenbach (Talk | Contribs) 01:30, 28 October 2009 (UTC)[reply]
That's going to be a tricky experiment for a non-professional to arrange. Persuading a statistically reasonable number of blonde and brunette girls (20? 50?), whom you've never met before (they can't be people you might recognise), to talk to you while you're blindfolded without telling them why (that's important - so as not to bias the results)...not so easy! If you're going to do it - make sure you control for age - and if possible, find some people who dyed their hair so you can figure out whether that's significant. I would be somewhat surprised if this were true - but it's perfectly possible and would make a great science project. But the real issue here is: "Can we find any pre-existing evidence?" - I couldn't find anything with Google or Wikipedia searches. SteveBaker (talk) 02:07, 28 October 2009 (UTC)[reply]
Well, let's consider natural color with the age between 18 and 30. However this depends on country, it should have a significant amount of girls with both hair colors to conduct trial. 94.20.22.147 (talk) 10:16, 28 October 2009 (UTC)[reply]
You would still have to correct for age. There is quite a lot of research on female voices, dominance and attractiveness but pitch lowers with age continually 18 to 30. The dominance thing matters too though. Both men and women naturally raise the pitch of their voice when there are people whose dominance they accept present and lower the pitch when asserting. You get taught this kind of thing in body language courses. To the extent that hair colour affects confidence affects assertiveness it is even possible hair dye will change pitch. --BozMo talk 11:55, 28 October 2009 (UTC)[reply]
You could go to Amazon Mechanical Turk and request people to enter their age, hair color, and a short audio clip of them saying a set phrase, without telling them what this is all about, and then listen to the recordings without letting yourself see the hair color they entered. Or to homogenize data, you could specify that only females in a certain age range are eligible for this HIT (Human Intensive Task). You'd have to give some small amount of money for each piece of data, as 10-15 cents is common.20.137.18.50 (talk) 15:02, 28 October 2009 (UTC)[reply]
I'm not aware of any experimental data that addresses this question. However, mechanistically speaking, I would suggest it is unlikely. There are some genes known that result in hair colour differences and none of them are known to be involved in the formation or function of vocal folds. Of course, it could simply be that no-one has studied them closely enough yet. But the idea that hair colour being correlated with other characteristics is not unheard of, for example, there is a known mechanistic relationship between hair colour and pain sensitivity (e.g. PMIDĀ 15731586, PMIDĀ 12663858 ). Rockpocket 18:18, 28 October 2009 (UTC)[reply]
Indeed - but it doesn't have to be genetic. If (for example) the "blondes have more fun" and "gentlemen prefer blondes" memes were influential - perhaps blonde females might subconsciously adjust their vocal range to somehow try to seem more child-like or something. I'm certainly not saying that this happens - or that this is even a viable mechanism - but it's perfectly possible that something learned in childhood or teen years that is culturally related to hair color has something to do with it - and hence, genes need not be involved. SteveBaker (talk) 22:45, 28 October 2009 (UTC)[reply]
No one yet has raised the issue of the correlation between age and dying dyeing one's hair blonde. My purely WP:OR wild guess as to what might be going on is: A very young (and hence high-voiced) dark-haired woman doesn't have much disposable income, and gets plenty of attention from men without much effort, so she isn't all that motivated to spend the time and money to get her hair dyed blonde. A somewhat older (and hence lower-voiced) dark haired woman has more disposable income, and is more motivated about trying to hold onto her fading youthful looks in whatever ways she can, so she's more likely to dye her hair blonde. Genetics has nothing to do with it. Red Act (talk) 23:19, 28 October 2009 (UTC)[reply]
Um, dyeing rather than dying, unless there's some deeper thing going on here.... --Trovatore (talk) 23:25, 28 October 2009 (UTC)[reply]
Whoops, that's one the spell-checker can't catch. Red Act (talk) 23:33, 28 October 2009 (UTC)[reply]
Steve's suggestion is certainly possible, however vocal range is mechanistically associated with structural characteristics of the vocal folds. One has the ability (subconsciously or not) to affect a change in vocal pitch, but only with a limited range. Beyond that range, one's pitch is limited by biophysics and anatomy. A serious study into this would be able to distinguish between affected and biological differences in tone. The problem with social or environmental effectors is that they would be near impossible to control (given the propensity and popularity of hair dye these days). Finding a genetic effect would probably be the only way to show a statistically valid difference, since you can dye your hair as often as you like, but your genotype always remains the same.
As for the issue of age, any statistically robust study would use age-matching (did the OP do likewise in his or her survey?) Rockpocket 23:49, 28 October 2009 (UTC)[reply]

plz help fft plots

Moved to Computing desk - Falconusp t c

Hard drive for archiving video?

I have some videos I want to keep long term,indeed beyond my lifespan.If I stored them on a hard drive recorder,computer or plug in hard drive and did not use the drive again, how long would it be before the video or other data started to degrade irretrievably, compared to DVD or Blu-Ray disks stored in the dark and not played for example?80.2.204.58 (talk) 10:58, 28 October 2009 (UTC)[reply]

After you are dead the chances are that those cleaning up after you will through them away. Your best chance would be through massive replication on many servers. Constant maintenance will be required to keep the file in a format that can be used, as your codec may not be down loadable in a few decades time. I would recommend a free open source codec like ogg. Store the software to play it and its source along with your video too. Graeme Bartlett (talk) 11:40, 28 October 2009 (UTC)[reply]
There really isn't a medium for stuff like that which won't either physically degrade or become so obsolete that it won't work anymore. For example: When I was a kid, about 50 years ago, my parents took massive amounts of footage of my sister and me using "Super-8" cine cameras. Now, to play back Super-8 is almost impossible - even if the film stock is still in good shape (which it probably isn't). There are a few specialist places that'll convert it to something else - but they are horribly expensive. Give it another 50 years and it'll require someone in a museum to play back Super-8 (consider the difficulties in replaying a 1900's piano roll recording using one of the myriad of piano roll "formats" - or recovering sound from a 1900's Edison drum recorder or a 1920's wire recorder). I have software that I stored on my Atari-ST computer - there is no way for me to easily get that back - even if the floppy disks have survived.
In my opinion, the best way is to keep it on the Internet on some large public server. If the company that runs it is large enough, they'll make backups and copy your data from old, obsolete hard drives onto whatever futuristic technology comes next as a natural part of their upgrade cycle. That doesn't protect you from the file formats becoming so obsolete that they won't play anymore - but it's probably your best bet at keeping your videos intact. Put them on YouTube and on WikiCommons and on as many other similar sites as possible - one might hope that when they eventually decide to change file formats, they'd go through their existing video collections and convert them to the new format. You are unlikely to be able to do that and keep them private through - and of course it's not a cast-iron guarantee - look what happened to GeoCities this week - but at least the folks on the Internet Archive will keep copies as long as they stay in business.
But who knows? The thousands of people who recorded their own performances on piano rolls probably thought their formats would last forever too. It's likely that only the very best "most important" piano roll performances ever been transcribed onto vinyl disks and from there onto CD's and from there onto MP3's. Who knows how many of those will fall by the wayside whenever the next great thing comes along?
The migration of things like music and video into bits and bytes probably gives them the best chance at longevity because bits and bytes can be copied endlessly without degradation - but that was true for piano rolls too - and standards shift and cultural change has not done so well for them.
SteveBaker (talk) 12:08, 28 October 2009 (UTC)[reply]
And, in fact, because they are on paper, the piano rolls will probably last a few hundred years, providing they are stored well. The same cannot necessarily be said for magnetic media at all. The problem with the piano rolls is the player; the problem with magnetic media (or plastic media) is actual physical degradation over time, plus the player.
You would think there would be a market for long-term data preservationā€”e.g. a company that stored your data (under whatever terms you wanted), and then promised to not only make lots of copies of it in different locations, but to every five years or so transfer it to whatever the new storage medium format is at the time, etc.
Obviously it would only last as long as the company did, though, and I guess that's actually probably where the worst odds lie. --Mr.98 (talk) 13:07, 28 October 2009 (UTC)[reply]

Maybe a good bet would be to persuade the BBC that it was worth archiving?80.2.195.144 (talk) 14:01, 28 October 2009 (UTC)[reply]

I would think piano rolls could be recovered easily enough. Just run them through an optical scanner. The rest is a routine software project. If no one has yet done this, could be a business opportunity. --Trovatore (talk) 18:56, 28 October 2009 (UTC)[reply]
Sure, for someone with a lot of technical background - some highly specialised software, a large-format scanner, etc. But that's what I'm saying - after 100 years, it takes the skills of a relative expert to recover the information. And it's a lot easier with something physical like a piano roll - there is a simple 1:1 correspondence between the holes and slots and the sounds. Try that 100 years after the event with a poorly-documented, horribly complicated binary video file - possibly burdened with DRM encryption and who-knows-what else. It's not that it can't be done - it's that it takes the services of a proper antique media restoration service - and that's never going to be cheap. In 10 years time, you can probably still replay it on any PC you have at home. In 20 years, you'll have to ask around friends - look for old computer clubs - maybe find an emulator. In 40 years, it'll be impossible to run that old software and you'll probably have to pay some company a pile of money to get it back for you. In 100 years - you'll need the services of a museum-grade expert - who'll probably only consider doing it for things that have historical merit and as a part of some bigger on-going research activity.
That's why putting it on the Internet with something like YouTube would be best - assuming the company you choose doesn't go bust (and you could place it with half a dozen companies to ensure against that) - they'll upgrade their hardware as needed - migrating existing files over onto the new solid-state drives - or the holographic laser storage unit - or whatever the next trendy mass-storage device turns out to be. They'll keep backups as needed - and if the format they use for storing video ever has to be superceded, you know that they won't just ditch all of the existing material - they'll write conversion software and convert to whatever new format is trendy. This incremental tracking of the technology is what will keep your files usable. However, this is still not a cast-iron guarantee - look what happened to GeoCities yesterday. SteveBaker (talk) 22:38, 28 October 2009 (UTC)[reply]

maximum frequency infinite or not?

Is maximum electromagnetic radiation frequency almost infinite (which means nearly an zero wavelength) or no limit with an infinite frequency and zero wavelength? Biggerbannana (talk) 12:43, 28 October 2009 (UTC)[reply]

The frequency must be finite. The electromagnetic radiation must consist of at least one photon, and the energy of a single photon is E=hĪ½, where Ī½ is the photon's frequency, and h is the Planck constant. So the frequency must be finite, because an infinitely large frequency would require an infinite amount of energy, which doesn't exist. Red Act (talk) 13:20, 28 October 2009 (UTC)[reply]
That's a cheap out, and not even certain - how do you know that an infinite amount of energy does not exist? A more interesting question would be if there is a theoretical limit to the wavelength of EM radiation. What if it becomes smaller than Planck length? What if the photon has enough energy to become a black hole? BTW, Biggerbannana, classical thermodynamics predicts unlimited energy photons from cavity resonators (or black bodies) - see Ultraviolet catastrophe. Einstein and Planck saved us from that... --Stephan Schulz (talk) 13:30, 28 October 2009 (UTC)[reply]
We know energy is not infinite because the universe behaves as though it has a finite amount of energy (or more properly energy/matter; since they are one in the same). If the universe had an infinite amount of energy, then it would have an infinite amount of free energy as well, and there would be no thermodynamic drive for work to occur. A photon could not contain an infinite amount of energy because then it would contain all the energy of the universe and then there wouldn't be any energy left for, you know, the rest of the universe. I suppose one could consider a pre-big-bang (or an instant-of-big-bang) something like an "all of the energy in the universe" particle, but in the current universe, once all the laws have settled out, such an idea makes little sense. --Jayron32 13:48, 28 October 2009 (UTC)[reply]
Sorry, I don't see how your claim follows. Our current models favor, but do not require an (unbounded) finite universe, but are compatible with a flat, open universe. In that case, total energy is almost certainly infinite (or average energy density would be zero). Yes, energy in the observable universe is certainly finite, but that's a different question. Anyways, the limits I suggested as possible are much stricter than "all the energy in the universe". Unless I miscalculated somewhere, a photon at Planck length would have 12.415GJ, the mass equivalence of 0.13 microgram. --Stephan Schulz (talk) 14:58, 28 October 2009 (UTC)[reply]
If a photon had an infinite amount of energy, it would form an infinitely large black hole. If such a thing existed in the visible universe, it would be pretty darn hard to miss. So a ridiculously pedantic answer might be more along the lines of "There is strong evidence that there does not exist a single photon with an infinite frequency, anywhere within the visible universe."
However, the OP looks to me like a straight-forward question, looking for a straight-forward answer. Bringing up infinite-energy black holes in response is a little like getting a question like "What insect is this?", and replying that there's a possibility that the photo might be of a robotic spy, carefully constructed to look like a insect by ancient inhabitants of a planet in orbit around Betelgeuse. That might be true, and that possibility might be fun to think about, but that's not really the kind of answer being requested. Red Act (talk) 15:35, 28 October 2009 (UTC)[reply]
I think we disagree about the interpretation of the question. It's not about an infinite energy photon - the question is if there is an upper limit to the energy of a photon. Each number in N is finite, but there is no upper limit. Your reply (there is only a finite amount of energy in the universe (debatable, but granted for this discussion), therefore the energy a photon can have is bound by this limit) is obviously correct, but it avoids the issue if there is a limit dictated by the laws of physics, not by the boundary conditions. If there is such a boundary, it could be much stricter than the one give by your argument. --Stephan Schulz (talk) 15:46, 28 October 2009 (UTC)[reply]
Any upper limit to the energy of a photon would break relativity, as there is a point of reference from which any given photon has any given amount of energy. ā€” DanielLC 20:05, 30 October 2009 (UTC)[reply]

The article Electromagnetic spectrum says various things about the upper frequency limit: Its wavelength is thought to be in the vicinity of the Plank length; in principle there is no limit; it is 2.4x1023 Hz (1 GeV gamma rays); but 2.9x1027 Hz has been detected from astrophysical sources. See GZK limit for a theoretical upper limit to cosmic ray energies. However it seems not every cosmic ray got the memo. Cuddlyable3 (talk) 17:07, 28 October 2009 (UTC)[reply]

Also, of course, cosmic rays aren't, and are not part of the EM spectrum. --Stephan Schulz (talk) 17:16, 28 October 2009 (UTC)[reply]

Actually what I was hinting at/searching for was the highest possible frequency/shortest possible wavelength to determine from a lay point of view what particle it might be associated with or become. At some point it seems clear that particles such as a neutron for instance must express electromagnetic resonance of a particular frequency or wavelength short of infinite frequency. Is it a gamma ray or a cosmic ray or what. (Excuse my rather boring lack of understanding.) Biggerbannana (talk) 17:15, 28 October 2009 (UTC)[reply]

You may find de Broglie wavelength interesting. --Tango (talk) 21:50, 28 October 2009 (UTC)[reply]

Maillard Reaction in Pasta Production

I am trying to find more information on Pasta Production, how would you prevent the product form changing colour. The article ā€œMaillard Reactionā€ did not provide enough information. Regards, Jeanette ā€”Preceding unsigned comment added by Jeanette de Lange (talk ā€¢ contribs) 14:27, 28 October 2009 (UTC)[reply]

When I make pasta, I don't subject it to heating and therefore there will be no Maillard Reaction. Why do you think pasta production involves the reaction? --Phil Holmes (talk) 14:36, 28 October 2009 (UTC)[reply]
(edit conflict with Phil Holmes) How are you making your pasta? Fresh pasta is basically uncooked dough, and dried pasta is just dried, uncooked dough. There is nothing in the making of pasta that is hot enough to cause the Maillard Reaction to occur. It's all done at room temperature. Likewise, most pasta is cooked via boiling, and since the Maillard Reaction occurs at much hotter temperatures than the 212F/100C that water boils at, it is does not occur during the cooking process either. The ONLY applications I could see where the Maillard Reaction may occur is in applications like Fried ravioli or in some baked dishes, like lasagna, but generally only around the edges of the lasagna caserole, where the noodles are exposed and out of the sauce. --Jayron32 14:40, 28 October 2009 (UTC)[reply]

Why ozone is over antartica?

We know that there is ozone hole(depletion of ozone) over antartica. And that becoz of CFCs emmitted by us.. my question is that "when we are polluting air above us then why ozone layer above Antartica is effected". I apologise for my english.ā€”Preceding unsigned comment added by 117.200.54.99 (talk ā€¢ contribs)

Does our article on Ozone depletion help? In short, it's because the chemical reactions that break down ozone are very much more efficient if polar stratospheric clouds catalyze the reaction. This type of clouds forms only at very cold temperatures. --Stephan Schulz (talk) 15:21, 28 October 2009 (UTC)[reply]
That's horrible.--Gilisa (talk) 17:09, 28 October 2009 (UTC)[reply]
What's horrible? His description was accurate and hard to put simpler. Regards, --ā€”Cyclonenim |Ā ChatĀ  18:03, 28 October 2009 (UTC)[reply]
Not his description, the idea that as the hole in the ozone layer is grown, earth get warmer, icebergs melt, and there is no chance for the ozone layer to repair itself as polar stratospheric clouds can't be formed. (I'm not an expert-but it sounds like thats what happening).--Gilisa (talk) 19:40, 28 October 2009 (UTC)[reply]
The ozone layer does repair itself, once the sun starts shining on it again. Graeme Bartlett (talk) 19:43, 28 October 2009 (UTC)[reply]
What do you mean?--Gilisa (talk) 19:45, 28 October 2009 (UTC)[reply]
  • Read Ozone layer#Origin of ozone for how ozone is formed (UV + 02 = 03). A hole forms when depletion happens faster than regeneration. ā€”Akrabbimtalk 20:05, 28 October 2009 (UTC)[reply]
  • You got the correlation the wrong way. Ozone is formed whenever oxygen is hit by the proper wavelength of UV light (i.e. when the sun is shining). It is destroyed by various processes, but in particular by those involving chlorine (from CFCs) and polar stratospheric clouds. Polar stratospheric clouds form when it is cold enough. That's why the ozone hole grows in winter (no sunlight, many clouds) and shrinks in summer (fewer clouds, and a lot of sun). Global warming will actually lead to a slight decrease in stratospheric temperature, but the effect should be negligible. And since we have phased out CFCs in the Montreal Protocol, things have slowly started to improve. --Stephan Schulz (talk) 20:13, 28 October 2009 (UTC)[reply]
Some facts:
  1. Ozone depletion happens to some degree over the entire planet - not just at the poles.
  2. Ozone in the upper atmosphere is created by UV light.
  3. There is much less UV light at the poles because they have no sunlight for maybe 3 months of the year - and much less sunlight than elsewhere in the world for the rest of the year.
Because they have less ozone to start with, the general ozone depletion over the entire planet affects the polar regions much more than anywhere else. So even though ozone-depleting gasses like CFC's may be denser over more occupied parts of the world, there is more than enough at the poles to remove all of the ozone and to create a hole. As others have pointed out, factors such as the temperature and the nature of polar clouds tends to exacerbate this problem.
Ozone is a weird problem. At ground level, ozone is a nasty poison and one that we go to great lengths to avoid making. In the upper atmosphere, it blocks harmful radiation and saves us from all sorts of nasty skin and eye conditions. This confuses some people. "Yeaaah Ozone! Boooo Ozone!"
It's worth mentioning that prompt action in phasing out CFC's and other ozone-destroyers has caused a significant improvement in the polar ozone levels - this is an encouraging thing since it suggests that humanity can get it's act together to fix global problems. Now if only we could do the same thing with CO2 and methane - we'd all be riding along on a much better planet.
SteveBaker (talk) 13:12, 29 October 2009 (UTC)[reply]

Electron configurations of rhodium and tungsten

Rhodium's electron configuration is [Kr]5s14d8, rather than the expected [Kr]5s24d7. While several other transition metals have similar anomalies, they have arrangements that either have two shells half-filled (such as chromium) or one shell half-filled and another shell full (such as copper). Rhodium's electron configuration doesn't fit either of these. Also, why is tungsten's electron configuration [Xe]6s24f145d4, when the same reasoning that applies to chromium should apply to it as well? ----J4\/4 <talk> 16:27, 28 October 2009 (UTC)[reply]

From your question it looks like you've read Electron configuration article already; please read it if you haven't. Here is a more detailed answer. In atoms of d-block elements, you have three possible ground configurations: (n+1)s2ndk, (n+1)s1ndk+1, and (n+1)s0ndk+2. These three configurations are split into LS terms, which in turn are split into states. The configuration-average binding energies for the three configurations are quite similar; the difference in binding energy between (n+1)s2ndk and (n+1)s1ndk+1 is usually smaller than the differences in binding energies between some of the terms of each configuration. So it may happen that the ground state (the strongest-bound eigenstate) belongs to either one of those configurations, depending primarily on the LS terms allowed for the given value of k by the Pauli principle. Also, binding energies depend substantially on the (fractional) probability to find an outer ("optical") electron close to the nucleus, where coulomb attraction is strongest. This probability is very different for different values of the nucleus charge (different elements), so the d-block elements in periods 4, 5, and 6 may not fill out their outer shells in the same order. For example, Niobium has ground state belonging to configuration 5s14d4, but Tantalum has 6s25d3 and not 6s15d4. Complicating matters further, the three configurations (n+1)s2ndk, (n+1)s1ndk+1, and (n+1)s0ndk+2 have the same parity, so the true eigenstates may be mixtures of states in those configurations; that is, electron wavefunctions are sums of components from different LS terms of different configurations. In such a case it is not even possible to unequivocally determine which configuration is indeed the ground one. --Dr Dima (talk) 19:20, 28 October 2009 (UTC)[reply]
(continuing - hit Save instead of Preview. Sorry). So for your first example, Cobalt is 4s23d7 but Rhodium is 5s14d8 and not 5s24d7. That is, very roughly, because the difference in Coulomb interaction with nucleus in Co (Z = 27) 4s and 3d orbital is not the same as difference in Coulomb interaction with nucleus in Rhodium (Z = 45) 5s and 4d orbitals. There are other effects at play, too: difference in electron-electron interaction terms, stronger relativistic effects in Rh than in Co, and so on. The bottom line: terms of 4s23d7 overlap with terms of 4s13d8 in Co, and terms of 5s14d8 overlap with terms of 5s24d7 in Rh; it just so happens that the lowest energy term is not in the same configuration in the two cases, for the reasons explained above. The same applies to Tungsten vs Chromium and Molybdenum, as well. --Dr Dima (talk) 19:36, 28 October 2009 (UTC)[reply]

Spectroscopy

I'm working on the bio of a chemist, Basil Weedon, I want to link to an article detailing what type of spectroscopy he used to elucidate the structures of carotenoids. The source says he used proton magnetic resonance but I can't work out what this actually is to link to, if someone can provide a link I'd to what this is called on here I'd be greatful. Thanks for your help. Smartse (talk) 18:19, 28 October 2009 (UTC)[reply]

Magnetic_resonance_imaging#Magnetic_resonance_spectroscopy. --Sean 18:31, 28 October 2009 (UTC)[reply]
In particular, NMR spectroscopy. It's the standard chemists' tool for determining molecular structure. I see that proton magnetic resonance has become a bluelink to an article with lots of details about the specific types of this analysis. DMacks (talk) 18:50, 28 October 2009 (UTC)[reply]

Planets shrinking

Is all planets lossing size in diameter or is just Mercury? Is Uranus and Pluto alos losing size in diameter?--209.129.85.4 (talk) 20:08, 28 October 2009 (UTC)[reply]

The article says: "3 miles of its 9,000 mile diameter in four billion years" - that's 0.001 of a millimeter per year - a REALLY slow rate of loss. I doubt we have good enough images of the outer planets to measure to that kind of precision. I'm skeptical that we can measure Mercury to that precision either! Whoever says this is true must have inferred it indirectly somehow. SteveBaker (talk) 20:32, 28 October 2009 (UTC)[reply]
I believe the surface has wrinkled as it shrank and that is how we know. I think the gas giants are shrinking, at least Jupiter is - from Jupiter: "This additional heat radiation is generated by the Kelvin-Helmholtz mechanism through adiabatic contraction. This process results in the planet shrinking by about 2 cm each year." --Tango (talk) 20:45, 28 October 2009 (UTC)[reply]
Yeah, found it - Mercury (planet)#Plains: "One unusual feature of the planetā€™s surface is the numerous compression folds, or rupes, which crisscross the plains. As the planetā€™s interior cooled, it may have contracted and its surface began to deform, creating these features." --Tango (talk) 20:47, 28 October 2009 (UTC)[reply]
The rate must be even slower than the linearly-interpolated 1 micron per year because as the planet cools, the litospheric thermal boundary layer increases in thickness and insulates the inerior. The ridges are thought to be signs of contraction, but this is an inference from Mariner images; we'll learn a lot more from Messenger soon. Awickert (talk) 02:46, 30 October 2009 (UTC)[reply]

IR spectrum of benzophenone

I will admit first off that this does constitute part of a non-assessed assignment. I have spent ages trying to work this out and I think I may just be having a blonde moment. Basically I have an IR spectrum of benzophenone with percentage transmittance on the y axis and I have managed to identify the origin of most of the peaks. However, there are three peaks around the 2000cm-1 mark. Specifically at; 1977.64, 2028.75 and 2159.88. Thank you for any help.Alaphent (talk) 20:20, 28 October 2009 (UTC)[reply]

Infrared spectroscopy correlation table may be helpful to you; though I admit I cannot find any relevent peaks in those ranges that would be expected from benzophenone. My best guess is that the peaks are due to the presence of an impurity of some sort, either left-over starting material or a decomposition product or something like that. --Jayron32 02:02, 29 October 2009 (UTC)[reply]
Here's a quick google hit for the spectrum: [1], and I don't see any significant peaks like the ones described. DMacks (talk) 02:09, 29 October 2009 (UTC)[reply]

Ok. Thanks, I'd found the quiz from the uni of colorado. It must be an impurity then. The guy who set this would do a thing like that just to be awkard. Alaphent (talk) 19:57, 29 October 2009 (UTC)[reply]

Are we Europeans all Jewish?

It seems that if you go back far enough, everyone is a descendant of everyone (more or less) alive hundreds of years ago. According to TV (huh!) Jewishness passes through your mother and is not extinquished even if you never follow the religious rights. Does that mean that everyone of European descent is technically Jewish? 92.29.91.83 (talk) 20:47, 28 October 2009 (UTC)[reply]

No. While if you go back far enough everyone is descended from everyone else (Identical ancestors point), you aren't necessarily related to them through the maternal line. You only have one female ancestor in each generation that is just through the maternal line (your mother, your mother's mother, your mother's mother's mother, etc.). Only if one of them is/was Jewish, or you've converted, will you be Jewish under that rule. There is a very good chance that that won't be the case. --Tango (talk) 20:52, 28 October 2009 (UTC)[reply]
That said, I expect there plenty of people that are Jewish under that rule without knowing it, just not everyone. --Tango (talk) 20:54, 28 October 2009 (UTC)[reply]
Where did you get from the knowledge that all Europeans have Jewish heritage? I know about recent genetical studies in European Iberian population which reveald about 20-30% of Iberian people are descendant from Jews (on their paternal Y heritage if I remember it right), probably of the Anusim (and additional 10% from Arabs). I also heard that many of Russian people in Moscow descendant from Jews-but still there is a great distance from this to what you just suggested. Also, keep in mind that if X precents of Europeans descendant from Jews no more than X/2 are of Jewish maternal heritage. It was estimated once that without the holocaust and the high rates of assimilation and with the reproduction rates of Jewish families during the 1930's Jewish people would count more than 90 million people by now. Before the mid 19 CE rates of intermarriage of Jews with non Jews population were very very low and assimilated Jews have the reproduction rates of non Jewish European families (i.e., they can't exceed their initial precent in European population this way). So you can make the simple calculation and understand that there is no storng basis for this kind of assumptions.--Gilisa (talk) 21:01, 28 October 2009 (UTC)[reply]
That depends very much on when you start considering people Jewish. If (just as an example) the Queen of Sheeba had some children with Solomon, and you accept him as Jewish, they, as part of the nobility, would have plenty of opportunity to spread their genes around. Similarly, I would assume that there would be quite some mixed descendants from the time of the Jewish disapora in Babylonia, some of which may have been in a position to have a lot of children. There is some evidence that Ghengis Khan sneaked his genes into 0.5% of the male world population in just 1000 years, and these processes typically are exponential. I wouldn't be at all surprised if 90% of Europeans have some Jewish heritage within the last 2000 years (and, of course, the same holds for many other people - Romans, Greeks, any of the people culturally and economically integrated in the Roman Empire...). That said: It's a lot easier for males to have a large number of descendants, so for the female line criterion, the chances are a lot slimmer. --Stephan Schulz (talk) 21:23, 28 October 2009 (UTC)[reply]
I agree, the vast majority of the European population will have some Jewish ancestors. That is very different to having Jewish matrilineal ancestors. It isn't just enough to have a female Jewish ancestor or for your mother to have a Jewish ancestor, it needs to be women all the way from them to you. --Tango (talk) 21:32, 28 October 2009 (UTC)[reply]
...and we haven't even begun to discuss the concept of being "ethnically Jewish," but that is whole 'nother can of worms. Bus stop (talk) 21:41, 28 October 2009 (UTC)[reply]
And not one worth opening. Ethnicity isn't a useful way to categorise people; it is extremely imprecise and even if you can define it it says nothing about genetics just culture. --Tango (talk) 21:46, 28 October 2009 (UTC)[reply]
I saw an interesting program a while back about how much more related we are than we typically give credit for. The program said that if you went back far enough to a human that had a reasonably large number of offspring, eventually practically everyone in the world is at least somewhat related to them. The example they gave was Cleopatra. She lived 2000 years ago and had a bunch of children. If you assume this is about 100 generations (average generational gap of 20 years) and go back that many generations assuming you have 2 unrelated parents each generation, you get 2^100 or 1.2676506E10^30 people, more people than have ever lived (and maybe ever will live), an absolutely preposterously impossible number for us to have that many ancestors. The conclusion is that we are related to everyone that was alive at that time and didn't have a lineage that died out. Similarly, everyone alive today has two possibilities: in a few thousand years their line will have either died out (no children or particularly unfortunate children) or will be an ancestor to practically everyone on the planet. If looked at from a suitable number of generations, our "family trees" are actually more of a diamond shape. Of course we are related to some historical figures much more closely than others. They occupy a more prominent place in the diamond... Or if we are sticking with the tree idea, the single person occupies many many more of the spots. I suppose this doesn't apply to people completely isolated from the "main body" of humanity until relatively recently. That would include Native Americans and Australian Aboriginals, at a first guess. TastyCakes (talk) 22:02, 28 October 2009 (UTC)[reply]
Indeed. The identical ancestors point for the whole human race is estimated to be around 5,000-15,000 years ago. If you exclude groups that are or were until recently isolated, then it is much more recent (although I can't find any figures...). --Tango (talk) 22:22, 28 October 2009 (UTC)[reply]
That's a nice article I didn't know about. I think it puts the idea much less confusingly than I did above... TastyCakes (talk) 22:28, 28 October 2009 (UTC)[reply]
Stephan Schulz, Ghengis Khan had thousends of women, his heirs had similar numbers of women and you must remember that world population size was much much smaller during his life time, making founder effect to be highly plausible. As for the Jewish people, most of them are descendant from the same ancient paternal ancestor who lived in the Levant about ~ 4000 years ago and this genetic cluster is still well distinguished from other Middle Eastren populations. Also, about 40% of Ashkenazi Jews are descendant from only 4 matrons who lived in the Levant about ~1500 years ago. While you can find this maternal heritage in very small precents among non Jewish populations in the present days Middle East, you virtually can't find it among non Jewish Europeans. So, it's highly unlikely that 90% of Europeans are descendant from Jews. Also, these processes are growing in exponential rise only when the average number of offsprings is higher than the average number of offsprings in the target population. Usually, I think, population genetics models don't use exponential rise.--Gilisa (talk) 22:15, 28 October 2009 (UTC)[reply]
I disagree. It is practically certain that everyone in Europe (indeed, everyone in the world with a couple of exceptions) has an ancestor that practiced Judaism, due to the mathematics outlined above. Of course, the contributions to a person's ancestry may be so small as to be genetically trivial. As pointed out earlier, the matrimonial line requirement changes the story considerably. TastyCakes (talk) 22:22, 28 October 2009 (UTC)[reply]
If we assume that the reproduction rate of assimilated Jews and non Jews in Europe was similar-and we have no reason to assume otherwise (infact, as assimilated Jews had free occupations and lived in big cities usually their rates of reproduction were lower than these of non Jewish rural populations in Europe) and that European population grow during the last 100 years in exponential rate then the relative genetic contribution of Jews to the European population remains the same. --Gilisa (talk) 22:32, 28 October 2009 (UTC)[reply]
But it doesn't matter if the assimilation rate was even almost completely zero. All it would take is one exception far enough back in time for a Jew to become an ancestor to all Europeans, and the chances of that not happening in the thousands of years Judaism has been practiced is zero. I am not saying that Europeans have mostly Jewish DNA, most will have almost negligible amounts for the reasons you are saying. But all will have some. Similarly (and perhaps more mind-bogglingly), it is absolutely certain that you are related to a Chinese person that lived 15,000 years ago. Even though genetic intermingling was very low due to cultural and geographical separation, it wasn't zero and hence Chinese DNA undoubtedly mixed with all other Eurasians' (and Africans') DNA over the past few thousand years. TastyCakes (talk) 22:38, 28 October 2009 (UTC)[reply]
To clarify the basic error in the question, it's true that if you trace back all of a person's ancestors, the number doubles every generation you go back, except for overlap due to multiple paths leading to the same ancestor. That means if you go back a hundred generations, you get a major fraction of the world as ancestors. But if you only count mothers, you only get one for each generation, so for a hundred generations back, you only get a hundred mothers. Looie496 (talk) 22:46, 28 October 2009 (UTC)[reply]

By the same logic are we all Muslem? And are all Muslems Jewish and all Jewish people Muslem? 92.24.25.252 (talk) 01:38, 31 October 2009 (UTC)[reply]

I thought that acrylamide was produced as a result of the Maillard reaction, but the Maillard reaction article does not mention it. Does it or dosnt it create acrylamide please? 92.29.91.83 (talk) 20:53, 28 October 2009 (UTC)[reply]

The Maillard reaction article does not mention it, see the page on acrylamide - this does clearly state that that it may be a by-product - not the main product - of the Maillard reaction. It may well be that the Maillard reaction product decomposes to acrylamide with more heat, but that does not make it a product of the Maillard reaction, but a product of a subsequent pyrolysis reaction. Ā RonhjonesĀ Ā (Talk) 22:45, 28 October 2009 (UTC)[reply]

What is the intermediate product please? 78.144.248.102 (talk) 10:32, 30 October 2009 (UTC)[reply]

Icons of Evolution

I'm about to rip my hair out.

I read the article on Icons of Evolution. Please glance over it. Note that in the first sentence the book is condemned as pseudoscientific. Later a sweeping claim is made that all biologists endorse evolution. Hopefully, even if you are an evolutionist yourself, you can see that that isn't proper for an encyclopedic article. I can provide names of at least three prominent non-evolutionist biologists -- Michael Behe, Dean Kenyon, and Phillip E. Johnson. Don't be silly like Auntie E. and say that just because I provide only three names means that there are only three.

This is my problem. While reading the article, I noticed that the article lacked one huge thing: information on what was said in the book! This article claims to be ABOUT Icons of Evolution, but has nothing on what's discussed in the book. Nearly everything in the article is criticism -- cited criticism, at least, but still it's a very biased article.

You might say, "Well, go fix it yourself." I tried. I don't like to be abusive, but Auntie E. has been downright incorrigible. I eliminated the word "pseudoscientific." I took out the sweeping claims discussed above. Under some chapter headings, I added a little on what was discussed in the book. Did Auntie E. even read what I wrote before deleting it under the pretense that it was not neutral? I honestly don't know.

Please talk with Auntie E. This shouldn't be a question of Intelligent Design versus Evolution. This article should, if you apply common sense, be a pro-ID article, because for heaven's sakes the book that is supposed to be discussed in it is pro-ID! Anyone who tries to clean up the bias finds Auntie E. flaunting neutrality rules in their faces. I wanted to uphold the integrity of Wikipedia. Instead I'm being browbeaten by a biased "editor" who seems, by her conduct, to be interested in nothing but safeguarding her evolutionist beliefs. --Thalia14 (talk) 21:12, 28 October 2009 (UTC)[reply]

I'm not sure that this is the right forum, why won't you take it to wikipedia's mediation? --Gilisa (talk) 21:17, 28 October 2009 (UTC)[reply]
(Johnson is not anything like a biologist, he's a law professor, argues about the philosophy of materialism, etc.) Anyway, I do agree that the article suffers from quite a bit of POVā€”it is nothing but rebuttals, aside from one list of Wells' "icons", which really is quite insufficient for a book, even one full of contentious and challenged statements. Anyway... in my personal experience, the best you can do is be calm, try to appeal to more high-minded folks. You are unlikely to have any success, thoughā€”Wikipedia's NPOV on such topics usually is the SPOV, even though it isn't supposed to be. Editing contentious Wikipedia articles is basically a game of seeing who is most persistent and who knows how to use the system to their aims better. It is a frustrating and un-fun experience, and the stakes are, in my opinion, not high enough to warrant it. It is one of the reasons I barely edit articles anymore. --Mr.98 (talk) 21:28, 28 October 2009 (UTC)[reply]
I suggest you compare it with the Featured Articles, Uncle Tom's Cabin and Jack the Ripper: The Final Solution. Notice how both of these tell the reader very early on whether the work is widely believed to be true or not. They both devote a good deal of space to discussing how the works were received, and how they are currently received, particularly by people who are experts on the topics the books are about. They also include brief information on the key contents of the works: you can find the same in the Icons of Evolution article under Wells' Icons. It's quite a large section.
You need to read WP:NPOV (again, if you already have) and understand that the article should not be pro-ID at all. If your real problem is that you think evolution can't possibly be true, and therefore anything which does not support Wells' criticisms must be dishonest, you need to step back and decide whether you can honestly edit any articles on this topic without getting emotionally involved and violating NPOV. 80.41.80.71 (talk) 21:41, 28 October 2009 (UTC)[reply]
I think that she made few good points. Exclusion of information from an article, when there is room for it, is second worse only to inclusion of wrong arguments in it. It doesn't matter what you believe in, objectiveness must be kept and I think that maybe in this case it was missing to certain extent. More, I understand that you didn't discuss it on the talk page? Also, I suggest you to take the advises of Mr.98. Also, remeber that you may use Wikipedia different boards if you feel that edits on the article are being done without agreement. I suggest you to seek mediation by third party.--Gilisa (talk) 21:49, 28 October 2009 (UTC)[reply]
Yes, a better place is the talk page, or WP:NPOVN, or a RfC. But I disagree with much of the criticism - starting with the small, I could not find the "claim that all biologists endorse evolution". --Stephan Schulz (talk) 21:56, 28 October 2009 (UTC)[reply]
You're both right the first thing to do of course would be to actually try discussing this on the article talk page. I see User:Aunt Entropy has indicated a willingness to do so and it looks like at least one other editor is as well. I suspect if the user actually attempts to discuss it, other people would be willing as well. However attacking other editors is not going to help the discussion so I suggest Thalia refrain from doing either of these if they wish to discuss it further in the talk page Nil Einne (talk) 22:36, 28 October 2009 (UTC)[reply]
This is absolutely not the right place to bring up these complaints. APL (talk) 22:24, 28 October 2009 (UTC)[reply]
Well well a disagreement between two Scientists called Steve on ending this discussion. I am all for ending it myself. It really is not about a scientific subject. --BozMo talk 13:04, 29 October 2009 (UTC)[reply]


May I suggest the Help Desk for editing help? 66.65.140.116 (talk) 00:12, 30 October 2009 (UTC)[reply]

Saturn V versus Ares rocket: Reinventing the wheel.

NASA and the TV news made a big deal of the launch of the new Ares rocket, going on about how amazingly big and powerful it is. Checking the stats, the Saturn V was 110.3 meters tall versus 94 for the Ares 1-x, had a diameter 10.1 meters of compared to 5.5 meters, and could deliver 118,800 kg to low earth orbit compared to just 25,000 kg. The Ares thrust is not stated in the Wikipedia article, but a website says it is "3 million pounds" compared to 7,648,000 for the Saturn. My question: Is the Ares less than 21% of the cost per unit compared to the Saturn V (inflation adjusted)? The remaining development costs are also to be considered for the new rocket. The Saturn also has a demonstrated safety record. Why didn't NASA dust off the blueprints for the old Saturn V and have Boeing build a few? (If the U.S, government threw away the detailed plans, they could probably get a copy of what spies likely supplied to the USSR.) Edison (talk) 22:49, 28 October 2009 (UTC)[reply]

They shouldn't be making a big deal about how big and powerful it is. That isn't the important thing about it. You don't just want the most powerful rocket you can get, you want a rocket that is right for the job you intend to use it for. Ares I is just supposed to carry the people up to LEO, Ares V will do the heavy lifting. (Assuming they actually get that far - that is rather in doubt at the moment.) --Tango (talk) 22:55, 28 October 2009 (UTC)[reply]
Hopefully this doesn't come across as soapboxing as it is largely OR. I read on one of the article talk pages (found it here Talk:Orion (spacecraft)#Falcon 9 Heavy to replace Ares I?) someone saying most people recommended NASA should have designed something like the Soyuz (after all the Chinese did and it appears to work well for both of them, the Russians and Chinese are fairly secretive about their safety record and costs but I would expect the US government has a rather good idea of both) but NASA ignored them because they didn't want to use something they didn't design. I don't know if this is true, but it does seem likely NASA would find it politically problematic to copy another country particularly the Russians. In the same vein, I would wonder whether just reusing a perhaps slightly modified old design would have similar problems of being not politically acceptable. After all when you have people like this [2]... (Okay the last bit was only of minimal relevance but I've been dying to use that since I came across it a few weeks ago) Nil Einne (talk) 23:36, 28 October 2009 (UTC)[reply]

Even if NASA wanted to build more Saturn V's, it wouldn't be just a matter of "dusting off the plans". Components that you could buy off the shelf in the 1960s are not necessarily available at all today, and that includes not only the parts used directly in the rocket, but also the parts used to make the devices used to make the rocket. For that matter, even the Vehicle Assembly Building is no longer available as long as it's needed for use with Space Shuttles. --Anonymous, 23:55 UTC, October 28, 2009.

To some extant they have: Apollo Heat Shield Uncrated After 35 Years, Helps New Crew Vehicle Design and Museums guide NASA to future missions Engineers study Apollo exhibits to learn how to return to moon:

"Snoddy, a manager at NASAā€™s Marshall Space Flight Center, has been removing valves and other parts from Apollo exhibits as he oversees construction of the upper-stage engine on the new moon rocket, dubbed Ares 1." Rmhermen (talk) 01:19, 29 October 2009 (UTC)[reply]

As Anon said, one of the main reasons they did not re-use the Saturn V is because the infrastructure for building it is not in place. NASA was not the only organization involved in the Apollo missions--they contracted out a lot of the R&D and construction to corporations around the country. All of the facilities, personnel, and resources that were used to build the Saturn V are now being used for other purposes that may not even be related to the space program (for the record, the Vehicle Assembly Building actually was used for the assembly of Ares I-X, it's the only building in the world that has that capability). The Constellation program actually does use a lot of old infrastructure, as the OP suggested--a lot of the hardware is based on the Shuttle program! Ares I is adapted from the Shuttles' Solid Rocket Boosters, and the liquid rocket of Ares V is adapted from the corresponding system on the Shuttle. Mildly MadTC 18:16, 29 October 2009 (UTC)[reply]

There was no existing infrastructure for creating pumps, combustion chambers, fuel storage tanks, etc for Saturn. It was all created in a very few years. This is a red herring. It could be re-created with the benefit of 20-20 hindsight. As I asked initially,, is the Ares proportionately cheaper to its lower earth orbit capability? Recovery of the solid rocket casings, compared to throwaway of the Saturn booster is not a convincing factor in favor of Ares, absent cost information. The ability of the Shuttle to land and take off again was supposed to make spaceflight super cheap, but it did not turn out that way. Edison (talk) 04:56, 30 October 2009 (UTC)[reply]

tiredness

why is it that at bedtime I feel sleepy but if i stay up half an hour past my usual bedtime I'm then not sleepy? ā€”Preceding unsigned comment added by 86.166.235.46 (talk) 23:16, 28 October 2009 (UTC)[reply]

Its probably a consequence of your circadian rhythm. If you maintain a regular bedtime schedule, its likely your body has become entrained to certain zeitgebers that define your bedtime. You then upset that schedule, your rhythm goes out of whack, and the molecules in you body that makes you feel sleepy are no longer there. Rockpocket 00:04, 29 October 2009 (UTC)[reply]
The MOLECULES in your body??? Which particular ones? What sort of answer is that? This is a non answer of the highest magnitude. ā€”Preceding unsigned comment added by 79.75.3.92 (talk) 02:20, 29 October 2009 (UTC)[reply]
As Looie496 correctly points out below, the molecular mechanisms involved in sleep are not yet fully understood. While melatonin is clearly a big player in chronobiology, its unlikely that it is primarily responsible for the observation the OP is asking about. Nevertheless, anyone who has experienced jet-lag can attest that circadian rhythm and the feeling of tiredness/alertness have a close relationship. Rather than speculate on the precise molecules involved, I preferred to offer the OP an general overview of circadian rhythm (with links) and how it would effect sleep. If s/he or anyone else would like a more detailed answer, with links to primary sources, then I would be happy to offer them. Other than that, I would note that my answer was orders of magnitude more constructive than the childishly petulant outbursts of a certain banned former editor. Rockpocket 18:49, 29 October 2009 (UTC)[reply]
The primary molecules involved in the management of circadian rhythms usually include Melatonin. The idea is that melatonin levels wax and wane, and your body's response to it waxes and wanes as well. If either your melatonin levels drop, or if you become tolerant to existing melatonin levels, you may "miss" the opportunity to fall asleep. --Jayron32 03:41, 29 October 2009 (UTC)[reply]
"This is a non answer of the highest magnitude." Actually, it was a rather informed answer that Rockpocket generously took the time to provide. Nobody tells us that we have to answer questions here. Falconusp t c 03:51, 29 October 2009 (UTC)[reply]
Well, it was probably a guess, actually -- the relationship between circadian rhythms and sleep patterns is not that precisely understood. Another possible answer is that the questioner tends to engage in behaviors that are more stimulating on the occasions when he misses his usual bedtime. The melatonin-based explanation is pretty unlikely -- there are much more direct interactions between the circadian system and the brain circuits that control sleep. Looie496 (talk) 05:00, 29 October 2009 (UTC)[reply]
Becoming aware that one is missing one's normal bedtime and possibly anticipating that getting up the next morning will be difficult is a stress situation where the nervous system may generate Epinephrine (adrenalin) hormone which is a "fight or flee" stimulant.Cuddlyable3 (talk) 10:38, 29 October 2009 (UTC)[reply]
You may be addicted to caffeine. If you abstain from caffiene then you will feel not so good for a few days, but then sleep very well and wake up feeling refreshed and alert all the time (as if you had just had a large coffee). 78.144.206.114 (talk) 11:27, 31 October 2009 (UTC)[reply]
Whoah there 78.144.206.114. You gave medical diagnosis and advice which we DO NOT GIVE. Cuddlyable3 (talk) 14:55, 31 October 2009 (UTC)[reply]


October 29

Effect of smokeless tobacco on dental health

What effect, if any, does smokeless tobacco (in general) have on dental health? Please answer from a dental practitioner's perspective, if possible. Thanks, ā™ The Ace of Spades(talk) 01:52, 29 October 2009 (UTC)[reply]

Wikipedia does have articles on smokeless tobacco, however looking though them, especially the one on Dipping tobacco, it appears the article has been entirely whitewashed by tobacco apologists. There are no actual links to any hard, independent studies into the health effect of dipping; the entire health section merely spends its time refuting studies which it gives no space towards reporting the results on. Sadly, Wikipedia may not be the best place to get accurate information on this. --Jayron32 01:58, 29 October 2009 (UTC)[reply]
Nevermind, managed to find something elsewhere, thanks for the tip on that. Though this may not be the place to discuss it, should the dipping tobacco article be tagged with anything because of the said whitewashing? ā™ The Ace of Spades(talk) 02:13, 29 October 2009 (UTC)[reply]
Although the overall lifetime risk of malignant transformation in chronic users of smokeless tobacco is <1%, smokeless tobacco is carcinogenic and in addition to being associated with squamous cell carcinoma, verrucous carcinoma is uniquely associated with the use of smokeless tobacco. Additionally, smokeless tobacco may lead to gingival recession and is a modifying factor in periodontal disease, halitosis, staining and a possible increase in dental caries. Harmful constituents are readily absorbed into the oral mucosa with which they are in direct contact. Oral submucous fibrosis, of similar pathogenesis, is related to the chronic use of betel quid/areca nut, used by people in/of India, Pakistan and surrounding areas of Southeast Asia. Of submucous fibrosis cases, approximately 10-15% show epithelial dysplasia and roughly 8% exhibit squamous cell carcinoma. (All info from oral pathology course lecture notes, NJDS) DRosenbach (Talk | Contribs) 02:37, 29 October 2009 (UTC)[reply]
Thanks so much, that's exactly what I needed. Better than what I found online. ā™ The Ace of Spades(talk) 02:44, 29 October 2009 (UTC)[reply]

Human Hearing Range in Nature

Are all the frequencies in the human hearing range (12Hz-20KHz) present in humans' habitable biosphere, apart from what's produced by recent technology? If not, why/how did humans evolve to be able to hear them? --CodellTalk 04:55, 29 October 2009 (UTC)[reply]

While lacking the time to find and cite sources, I think it is safe to say yes, all frequencies between 12Hz-20KHz occur in the natural world humans live, and lived in. Pfly (talk) 09:32, 29 October 2009 (UTC)[reply]
Only a small range of frequencies about 300Hz - 3kHz is really needed to communicate using speech (e.g. used in telephones) and human hearing has evolved greatest sensitivity in this range, arguably because of the survival value of a parent hearing the cry of their baby.Cuddlyable3 (talk) 10:29, 29 October 2009 (UTC)[reply]
Greatest sensitivity maybe in that range, but a telephone quality sound is hardly high. Perhaps the ability to hear higher frequencies results from these other frequencies being present as harmonics? I have no evidence to back this up its just a thought. ā€”Preceding unsigned comment added by Alaphent (talk ā€¢ contribs) 14:25, 29 October 2009 (UTC)[reply]
Hearing higher frequencies has survival value because they aid localization of sound sources. Many sounds have harmonics beyond the human hearing range, the upper limit of which drops way below 20kHz as one ages.Cuddlyable3 (talk) 19:57, 29 October 2009 (UTC)[reply]
The sound of the ocean coast is an extremely good source of white noise which contains that whole range of frequencies. I'm thinking wind in a forest would too, maybe wouldn't go as low as ocean but it would probably still pass. Vespine (talk) 21:43, 29 October 2009 (UTC)[reply]
So maybe being able to hear the white noise of the coast and wind through trees would be beneficial for humans as both were a source of food. Might cultures who have lived for a long time in desert regions therefore have a lower upper hearing range? --CodellTalk 00:09, 30 October 2009 (UTC)[reply]
Well first of all, humans didn't actually evolve hearing so I think that's where your first misconception lies. Hearing has been around a long long time since before humans so it's quite probable we just inherited our range from our ancestors. Also, don't fall into the trap of thinking that every single observable trait in every species must have been the result of some selective pressure or evolution. Amongst others there are traits which are vestigial which are seemingly useless traits left over from past evolution, like goose bumps, if there is no real pressure to get rid of the trait there is no reason for it to disappear if it is not very costly to the organism, and also there are associated traits which can also seem useless but are somehow genetically linked to other seemingly unrelated traits which do have a good use. Vespine (talk) 04:28, 30 October 2009 (UTC)[reply]
Good point. I'll also note that hearing higher frequencies could have survival value because it helps us discriminate between different sounds. You can tell a violin from a trumpet when they play the same note because the overtones are different -- the same might apply to sounds in nature that it's desirable to tell apart. --Anonymous, 19:09 UTC, October 31, 2009.

Counter circuit Photo-Electric Beams

I want to design a counter using laser beams. The purpose is such that if someone enters the room they are added to the list. If they move out the it they are removed from the count. If the count is 0 then it switches off the lights, if it is more than 0 it switches on lights.--yousaf465' 08:11, 29 October 2009 (UTC)[reply]

You might construct your counter by soldering some logic integrated circuits on a small circuit board. The counter itself could be a 4516[3] which being 4-bit limits the count to 15 7 persons. If you need to store larger numbers of persons use more counter stages. Detecting whether a person comes in or out needs two photocells and lasers. With a 4013 dual flip-flop you can store the first photocell pulse then the second pulse clocks the counter and the order of the pulses determines the count direction up or down. The empty=0 state of the counter controls the lights. This logic circuit runs from say 5V so it needs interfaces to the small signals from the photocells, possibly made with op-amps[4] like 741Operational amplifier, and to the lamps. If the lamps are mains powered then you need an optoisolator[5]+triac[6] or a relay to control them safely from your 5V logic signal. When the counter is full=15 7 or empty=0 it must be disabled from counting further in the relevant direction. The references give more information and there are many alternative components.Cuddlyable3 (talk) 10:23, 29 October 2009 (UTC)[reply]
Thanks for the answer. I will look into it.--yousaf465' 12:46, 29 October 2009 (UTC)[reply]
The usual way to achieve this is to install a cheap passive infrared sensor in the room rigged up to switch off the lights if it detects no movement for, say, seven minutes. You find such systems in public buildings etc, so they must be available off the peg.--Shantavira|feed me 16:00, 29 October 2009 (UTC)[reply]
What if I want to sit alone in the OP's room and read a book for 8 minutes? Cuddlyable3 (talk) 19:52, 29 October 2009 (UTC)[reply]
Then every 7 minutes you wave your hands around like a maniac when the lights go off. (It's how it is done in universities, which often have such features.) --Mr.98 (talk) 00:40, 30 October 2009 (UTC)[reply]
This will help with exercises. --yousaf465' 02:52, 30 October 2009 (UTC)[reply]

Please see my circuit below. I limited the counter to 7 persons so that Q"8" limits the up count at 7, and CARRY limits the down count at 0. Cuddlyable3 (talk) 21:09, 30 October 2009 (UTC)[reply]

I'm sorry to say that what you propose won't work. With just one beam you can't tell whether the person is arriving or leaving - and you can't tell when someone steps into the beam - changes their mind and steps back again. You need at least two beams - then the order of breaking and unbreaking of the beams will tell you which direction the person crossed the beam - and whether they changed their mind again. But this is still not perfect - when one person enters at the exact same time that another person leaves - you can't know what happened.
At the heart of the problem here - if your counter EVER gets confused - even once - it'll either turn the lights off when one person is inside the room (and be extremely difficult to pursuade to turn them back on again) - or it'll turn the lights on with nobody in the room and refuse to ever turn the lights off again.
The difficulty with such systems is that you are trying to measure an absolute quantity (the number of people in the room) using a relative measurement (the number of people entering and leaving) - and any system like that that has any possibility of error whatever will gradually accumulate error until it's useless. In your case, it'll happen rather quickly...the very first time two people pass in the doorway for example. Even if you could sort out that case, you also have the problem if the power fails and your system has to restart - because you have no idea how many people are in the room to start with.
So at a minimum, you'll need a switch to say "Stoopid computer - you turned the lights off and I'm still here" or "Hey - turn the lights off, there's nobody in here!" - and perhaps some logic to (for example) turn the lights off after they've been on for a very long time. At least that provides some means for the system to recover from occasional errors.
The simplest way to implement the complex logic required would be to use a little 'Arduino' computer - you can buy one all ready to run on a circuit board for $25 - and if you can make your own circuit-board, you can buy just the chip for less than $5. You can write simple programs for it on your PC in C/C++ and download them into the computer using a USB port. With software involved, the complicated rules about breaking and unbreaking beams would be easy to set up. You could do timing and add software to try to decide when things have gone astray. You could even add a photosensor to turn the lights off during the day. You could do it with hard logic components - but you'll never do that for less than the cost of an Arduino chip.
A much, MUCH better solution would be something like a burglar alarm - something that detects motion or body heat or sound or floor pressure or something. Those are absolute measurement devices - not relative like a "number of people who went through the door" detector. Of course a cat can fool a motion detector, leaving the TV turned on in the room can generate enough heat to fool a heat sensor - ditto with sound detectors - and moving the furniture around can fool a pressure sensor.
This is actually a really tough problem to solve - which is probably why such systems aren't fitted to every home on the planet.
At our offices at work, we have a RFID tag system that does that. You have a little plastic tag that you put in your wallet - it gives you access to the front door and to the elevator - and when you're within range of any of the detectors scattered around the offices, the lights in the area come on. When you aren't, they go off. If nobody is near a detector, the airconditioner turns off too. It works fine - just so long as you don't lose your RFID tag. My car (a MINI Cooper'S) has a similar system. You just need the circular "remote" (it's not really a "key") in your pocket and it unlocks and locks - and allows the engine to be started with a simple push-button - and it'll turn off the radio and the lights and lock the doors when you get more than a few feet from the car.
Having people carry something that can be detected uniquely at a distance is the answer.
SteveBaker (talk) 12:53, 30 October 2009 (UTC)[reply]
Yes I was also looking for a 2 beams system. I will upload the diagram how I visualized it. Also for washroom we can even eliminate the counter, after all you can't have more than one person in a washroom.Ā :)

Well for power I will be using rechargeable batteries.

I'm also interested in RFID but for my cat. How much that system cost. --yousaf465' 04:40, 30 October 2009 (UTC)[reply]

Any Fritzing diagram could be helpful.--yousaf465' 05:06, 30 October 2009 (UTC)[reply]
The problem isn't going to be fixed with two beams - or three or four for that matter. There are just to many ikky cases that can cause your system to miscount - and it only has to miscount by ONE and it'll be wrong forever more (unless you reset the machine somehow). Because people don't neatly cross beams like that (they may break it first with one arm, then their body and then the other arm, for example - or someone may wave their hand through the beam to see what it does - or they might be wearing a thick, fuzzy sweater that gradually breaks the beam causing a lot of rapid on/off pulses that could easily be miscounted. Any one of these behaviors (and worse-still, any of the two dozen others that we haven't thought of) will cause at least a single miscount - and that's enough to leave you sitting alone in the dark - or having your light left on 24/7. It's not just a tiny problem that you can just blindly ignore - it's the total downfall of the entire proposal. Getting rid of the counter for a single-occupancy area doesn't help that - all it is act as a one-bit counter...it can still get confused and once it's in the wrong state, there is nothing to correct that error. In short: IT WON'T WORK! SteveBaker (talk) 12:52, 30 October 2009 (UTC)[reply]
What about any system which works.--yousaf465' 13:01, 30 October 2009 (UTC)[reply]

A computer with a webcam and face recognition software (or perhaps recognition of more general features such as clothing or height) could be made to recognise individuals, ambiguous entry and exits and count them correctly. Trevor Loughlin13:39, 30 October 2009 (UTC)

That'd be a rather clear-cut violation of the KISS principle. Red Act (talk) 15:18, 30 October 2009 (UTC)[reply]

What I think would work great would be a motion sensor in combination with a one-beam doorway sensor. The motion sensor should be positioned such that it doesn't quite cover the doorway. The system keeps the lights on, unless the doorway sensor was triggered more recently than the motion sensor, and it's been at least 7 minutes since the doorway sensor was triggered.

A system like that would be better than just a motion sensor, in that it isn't necessary to at least wave your arms every 7 minutes in order to keep the lights on. And it's better than just a doorway sensor, in that there's no count maintained that could easy get off by one.

With a two-sensor system like that, if the system does wind up in an incorrect state, it's easy to fix without needing any additional user interface. If the lights are off incorrectly, because you've been sitting motionless in the room for seven minutes after someone else left the room, you just wave your arms to turn the lights on and put the system back into a correct state. Or if the lights are on incorrectly, because someone thought it'd be funny to leave the room by crawling under or stepping over the beam, you just walk into and out of the room to put the system back into a correct state, and the lights will go out seven minutes later. Red Act (talk) 14:47, 30 October 2009 (UTC)[reply]

I agree that's a better approach - you need some kind of absolute way to determine if there is someone in the room. Incrementing and decrementing a counter is not the answer. The problem is that humans are tough to recognise electronically. A curtain blowing in the wind from a partially opened window - or a rotating ceiling fan will trick most motion detectors. Pets will also trigger them. I agree that using a beam-breaking system in conjunction with the motion detector will work much better than either of them alone - but I strongly suspect you'd still find the light turned on when it shouldn't be a bunch of times. The only approach that I'm aware of that's used 'for real' is the RFID tag system. That (in effect) modifies the concept of "what is a human?" to be something that computers can easily understand. Of course it's still gonna fail if everyone keeps their RFID tag in their wallet - and then leaves the wallet on the table when they leave the room. SteveBaker (talk) 16:33, 30 October 2009 (UTC)[reply]

Approach #1: Treat the OP with WP:AGF. He wants laser BEAMS. Approach #2: Tell the OP they don't really want what they say they want and instead try to sell them these. I'm doing Approach #1, see image. Cuddlyable3 (talk) 21:02, 30 October 2009 (UTC)[reply]

Room Occupant Counter Cuddlyable3 (talk) 21:02, 30 October 2009 (UTC)[reply]
Well I would like anything which works, not only laser beams. Red Act's approach could be more useful if a simple diagram could be shown.--yousaf465' 04:06, 31 October 2009 (UTC)[reply]
In that case install a false floor resting on rubber blocks. When someone is in the room their weight depresses the floor that operates a microswitch that turns on the light. Cuddlyable3 (talk) 14:51, 31 October 2009 (UTC)[reply]
Well I don't want my floor to be damaged. I have to do it practically.--yousaf465' 13:13, 1 November 2009 (UTC)[reply]

Raisin aversion

There are cereal mixtures with "no raisins added". Food aversion (exception allergic reactions, toxicity) are socially transmitted (e.g. food taboos). Is there a biological reasion why some people dislike raisins? --Grey Geezer 11:39, 29 October 2009 (UTC) ā€”Preceding unsigned comment added by Grey Geezer (talk ā€¢ contribs)

Wait. Do you really think "disliking raisins" is the same as a "food taboo"? I think you are mixing raisins with apples here. If you instead want to ask "why do different people have different likes and dislikes (including food)?", it all ends up imho in a "nature vs. nurture" debate. Some genetic differences might be present (e.g., in olfactory receptor equipment of the individual in the case of food), but the individual experience also has an impact of what we learn as pleasant or unpleasant, in an often unpredictable way, for example, because subconscious perception can influence our "wiring" in the brain without us ever knowing. --TheMaster17 (talk) 12:19, 29 October 2009 (UTC)[reply]
Food aversion is not just socially transmitted. Variation in food preference is part of behavioural diversity which is a portfolio evolutionary strategy, especially when you are not talking about main staple diet. It is not obvious in evolutionary terms that raisins pose no risk to people, after all they pose a risk to dogs see Grape and raisin toxicity in dogs, are stale food and look like animal droppings. So some of us being programmed not to gorge them on sight is perhaps safer for humanity than if we were all inclined to love them to bits. --BozMo talk 12:22, 29 October 2009 (UTC)[reply]
It was not about "disliking" it was about "aversion" ("I can't eat that!"). Found another article where there was an association with "an animals teat" (but how many children have actually seen an animals teat?). So I settle for "Looks actually like a ... naah! I can't eat that!". Case closed. Grey Geezer 13:10, 29 October 2009 (UTC) ā€”Preceding unsigned comment added by Grey Geezer (talk ā€¢ contribs)
One of the most extensively used experimental procedure in memory study (at the molecular level) in rats is CTA. It appears that if rat experienced physical bad feeling after tasting new food it will never taste it again, as long as the feeling starts in no more than 8 hours delay from eating it. This is also the answer for why seek rats avoid eating, i.e., it will make wrong connection between good food and its seekness. And this is also the reason for which rat poison is slow working --Gilisa (talk) 13:13, 29 October 2009 (UTC)[reply]
We humans are so smart. Bus stop (talk) 13:20, 29 October 2009 (UTC)[reply]
Nope, it take us more times to develop CTA.--Gilisa (talk) 13:27, 29 October 2009 (UTC)[reply]
What does "CTA" stand for? Bus stop (talk) 13:34, 29 October 2009 (UTC)[reply]
Contidioned Taste Aversion.--Gilisa (talk) 13:37, 29 October 2009 (UTC)[reply]
Wow, we already have an article on Conditioned taste aversion. Bus stop (talk) 13:40, 29 October 2009 (UTC)[reply]
With a lot of the up to date information missing. --Gilisa (talk) 14:10, 29 October 2009 (UTC)[reply]
It could be due to adaptive behavior like CTA. As for a specific raisin mechanism, I highly doubt it's known. Food aversion has been studied quite a bit, but a true mechanism still isn't clear generally. It's been studied in many animals, and in humans with a wide range of conditions like autism (where it's very common), anxiety (PMID:15576070), and even after certain surgeries (PMID:16925376). This all indicates what you might expect, that there is a physiologic reason. - Draeco (talk) 13:49, 29 October 2009 (UTC)[reply]
Actually, we do know that blocking specific dopamin receptors would prevent the development of CTA. However, what we yet don't exactly understand is how the retrieval mechanism of CTA works (but even here we prograss in huge steps - Yadin Dudai's studies from 2007 and on are focused on experimental extermination of already acquired CTA) .--Gilisa (talk) 14:20, 29 October 2009 (UTC)[reply]
Hello Gilisa, thanks VERY MUCH for this piece of evidence. This could explain, why a tasty nutrient is refused (because there was an unpleasant experience along with it.) Thanks again! Grey Geezer 14:26, 29 October 2009 (UTC) ā€”Preceding unsigned comment added by Grey Geezer (talk ā€¢ contribs) [reply]
For nothingĀ :)--Gilisa (talk) 14:38, 29 October 2009 (UTC)[reply]
Sure. If you don't like raisins than you can be sure that they contain compounds that do not agree with your biochemical system. Vranak (talk) 14:52, 29 October 2009 (UTC)[reply]
It's enough that you only change, or even camouflage, the compounds that are responsible for the flavor of raisins for one to eat them without feeling any dislike. Once the food is within your body it's much harder for it to decide what it's, the only thing your body can tell at this stage is whether this food have nutritious value and how it effect on visceral feeling.--Gilisa (talk) 15:04, 29 October 2009 (UTC)[reply]
There's a very simple reason why some people dislike raisins: they are quite bitter. Their sweetness makes this hard to recognize, but people who especially dislike bitterness will still be sensitive to it. Looie496 (talk) 16:28, 29 October 2009 (UTC)[reply]
And here you suggest that humans are genetically programed to prefer certain tastes with individual differences. That's another good option for why our friend dislike raisins.--Gilisa (talk) 17:06, 29 October 2009 (UTC)[reply]

We have a voluminous article Taste that links to Acquired taste but not to Conditioned taste aversion. I like the smart human Bus stop did not recognize the abbreviation CTA. That seems an unhelpful addition to this alphabet soup that could be avoided by merging the article on aversion into a section here titled "Acquired distaste". Cuddlyable3 (talk) 19:34, 29 October 2009 (UTC)[reply]

Some people get quite ill if they eat fructose beyond a certain proportion to glucose consumption. Eat a lot of fruit and get stomach pain, diarrhea, fatigue and depression. Large amounts of raisins (or apples, fruit juice, honey, or corn syrup, etc. ) would be bad for such persons. See Fructose malabsorption. It is not a matter of taste preference so much as a metabolic incapacity. Edison (talk) 04:49, 30 October 2009 (UTC)[reply]

Earth spinning

There's no "great power" actively holding the Earth spinning around itself and orbiting the Sun at the exact same speed for eternity, right? It just happens to be at the current speed because of various physical properties such as kinetic energy. Is this right? If so, then is the Earth's speed actually slightly changing? If yes, by how much? JIP | Talk 19:32, 29 October 2009 (UTC)[reply]

The great power is gravity and if the sun and earth were solid masses orbiting in a vacuum then Kepler's laws of planetary motion and the Earth's inertia would keep the system running "for eternity" with energy conserved. In reality the sun and earth are not solid masses. Both tides on Earth and the Sun itself expend heat energy and the Earth/Sun system is not isolated from external disturbances such as asteroids and comets. So nothing is forever. Cuddlyable3 (talk) 19:47, 29 October 2009 (UTC)[reply]
You'll want to read Earth's rotation. The speed of the rotation has definitely changed over the years due to Tidal forces from the Moon. ~ Amory (u ā€¢ t ā€¢ c) 19:50, 29 October 2009 (UTC)[reply]
For numbers pertaining to the Earth's slowing rotation, see Tidal acceleration#Quantitative description of the Earth-Moon case. The Earth's orbit around the sun is also slowing down; see Year#Variation in the length of the year and the day. Red Act (talk) 20:08, 29 October 2009 (UTC)[reply]
It seems like you already got your answers, but just to put it in simpler words: As for earth spinning itself, the law of Angular momentum preservation keep body spinning around itself (around its center of gravity) as long as angular force is not operated on it and its shape remain constant.
As mentioned here already, tiadl forces against the direction of earth spinning are operating on earth so the spinning is being decelerate by 2.3 miliseconds once in 100 years.
Amory already mentioned that as for Earth orbiting around the sun - body would keep moving in direct line and in constant speed as long as other force is not operating on it and Earth orbiting the sun according to Kepler's laws of planetary motion.--Gilisa (talk) 10:52, 30 October 2009 (UTC)[reply]
Taking into account general relativity, even an isolated two-body system will lose energy as gravitational waves - but as you can read in the article, that's indeed very little for the Sun-Earth system. The current changes in the orbital parameters (i. e. their first derivatives with respect to time) you can find here. JPL's solar system dynamics site provides other interesting data as well. Icek (talk) 20:20, 30 October 2009 (UTC)[reply]

How many buildings 2 story and above are there in the U.S.A.

I would like to know how many buildings taller than 2 stories are there in the U.S.A. Completed and/or under construction, if at all possible. But mostly how many buildings in all 2 stories and above. Thank You Very Much. --76.122.225.119 (talk) 21:03, 29 October 2009 (UTC)[reply]

I very much doubt you will get a good answer to this. Two-story is a very common configuration for houses and there are a lot of those. Also planning and approval is a local government duty (and some few places isn't always required), not federal or even state. Rmhermen (talk) 00:56, 30 October 2009 (UTC)[reply]
If good estimates of this do exist, they are probably hidden somewhere in the US Census Bureau - Manufacturing, Mining, and Construction Statistics site. (E.g., here is a table of new residential construction from the 1970s-present that breaks things down into storiesā€”not quite the same thing as asked but it does give an estimate of the order of magnitude in regards to residential buildings in particular.) --Mr.98 (talk) 01:35, 30 October 2009 (UTC)[reply]
How accurate do you need? There are 300 million people in the USA, so I'd guess there are more than 5 million such buildings and less than 100 million. Dragons flight (talk) 02:00, 30 October 2009 (UTC)[reply]
To start, I tried Wolfram Search for "how many buildings in the US" and it didn't understand. (By comparison, it does answer how many people and how many dogs in the US, and includes a graph with the former question. The dogs get no graph.) Tempshill (talk) 03:15, 30 October 2009 (UTC)[reply]
If you did use the above link for new residential construction, you're still stuck with the problem of figuring out how many of those structures are still standing. Dismas|(talk) 04:57, 30 October 2009 (UTC)[reply]
Further, you have to define a building. Is a duplex a single building based on this question? There are apartments, condos, etc... We haven't touched on commercial and industrial buildings. Those can be complicated. Consider downtown Charleston, SC. Because of lack of space, it is common for the area (a wide alley) between two buildings on King Street to be filled in with another building. The result is a continuous wall along the street, but it is technically two buildings with a building crammed in between them. Is that three buildings or two buildings or one building? Further, I've been in a house there that is actually two houses that were close enough to be joined together. Is it one house now? -- kainawā„¢ 12:59, 30 October 2009 (UTC)[reply]
A house which is joined to another house is called semi-detached, they are very common in the UK. Lots of houses joined together side by side is a terraced house. Personally, I would define all those houses to be part of one building. I think that is how it is usually referred to in the UK. --Tango (talk) 00:46, 31 October 2009 (UTC)[reply]
The U.S. Census lists 78 million (+/- 0.5 million or so) houses with two or more stories compared to 41.5 million one-story ones.[7] But that doesn't include commercial buildings and if we had a figure for commercial buildings we would have to figure out how many were mixed use and counted twice. (also 78 million plus 41.5 million equals around 120 million - but the survey starts out telling us there are 128.2 million homes (+/- 48,000) so apparently 8 million houses have no floors?) The data is based on a survey of 53,000 addresses every other year. Rmhermen (talk) 13:35, 30 October 2009 (UTC)[reply]
I have seen this question on Ref Desk before, and the discussion there might be helpful, if a link to the appropriate archive could be found. Edison (talk) 15:38, 30 October 2009 (UTC)[reply]

Glycogen Storage in Human Muscle Cells

Hello. Where and how do the breakdown products of stored glycogen enter the cellular respiration pathway? Is glycogen stored in the vacuoles of the human muscle cells? Thanks in advance. --Mayfare (talk) 21:30, 29 October 2009 (UTC)[reply]

Its stored in bananas. 188.221.55.165 (talk) 22:10, 29 October 2009 (UTC)[reply]
Read the article on Glycogenesis. Glycogen is the form in which muscle cells can store glucose for immediate use. In the human brain it also play major role in cases where oxygen levels are very low and there is no other option for neurons to produce the energy they need to survive.--Gilisa (talk) 22:32, 29 October 2009 (UTC)[reply]
Perhaps glycogenolysis would be more apropos. DRosenbach (Talk | Contribs) 23:23, 29 October 2009 (UTC)[reply]
You are right. I didn't read his question correctly.--Gilisa (talk) 09:09, 30 October 2009 (UTC)[reply]

Does glycogenolysis occur in the vacuoles of human muscle cells? --Mayfare (talk) 01:57, 30 October 2009 (UTC)[reply]

Yes.--Gilisa (talk) 09:09, 30 October 2009 (UTC)[reply]

Ladybirds hibernating

Anyone know why ladybirds (or lady bugs if you are from the other place) hibernate in big groups rather than singly? It cannot reduce the risk of predators and I don't see warm as helpful but there are a few corners in our house which each year get several hundred bunched all winter all of which crawl off in the spring. Surely separate hibernation for a non colony insect makes more sense? Whats the advantage? --BozMo talk 22:25, 29 October 2009 (UTC)[reply]

For Coccinellidae being conspicuous is associated with reduced, rather than increased, predation; see Aposematism. --Dr Dima (talk) 23:12, 29 October 2009 (UTC)[reply]
Its because they secrete aggregation pheromones when they find a suitable overwintering site. Its not known how or why this happens, but it might be something to do with marking places that served them well the previous winter, on the basis that if the insects survived the winter there last year, they may do so again. (See also Insect winter ecology and Michael Majerus' Ladybird bible, ISBN 0-00-219935-1) Rockpocket 23:19, 29 October 2009 (UTC)[reply]
"Ladybugs practice communal hibernation by stacking one on top of one another on stumps and under rocks to share heat and buffer themselves against winter temperatures." Bus stop (talk) 23:29, 29 October 2009 (UTC)[reply]
Despite what our article states, its not quite as a simplistic as them aggregating together for warmth, since that could happen anywhere. Instead there appears to be a selection for special overwintering spots, suggesting there is location specific aggregation going on (see Pettersson et al, Eur. J. Entomol. 102: 365ā€“370, 2005). Its been hypothesized that a non-volatile compound, 2-isopropyl-3-methoxypyrazine, is responsible (Abassi et al Cell. Mol. Life Sci. 54: 876ā€“879), which additionally ensures that males and females are in the same locale when the breeding season comes around in Spring. Rockpocket 23:46, 29 October 2009 (UTC)[reply]


October 30

Collar-size of the woolly mammoth

Ok, I want to put a 16th century ruff about the neck of a woolly mammoth. How large would it have to be? Here's the dimensions from the WP article: they were not noticably taller than present-day Asian elephants, though they were heavier. Fully grown mammoth bulls reached heights between 2.8 m (9.2 ft) and 4.0 m (13 ft); the dwarf varieties reached between 1.8 m (5.9 ft) and 2.3 m (7.5 ft). They could weigh up to 8 tonnes.

Thanks Adambrowne666 (talk) 00:57, 30 October 2009 (UTC)[reply]

Is there some wooly mammoth neck diameter-mass-height ratio algorithm you know exists, but just don't know what it might be? Maybe figure one out based on the present day Asian elephant and fudge it -- ruff oglers will never know! DRosenbach (Talk | Contribs) 01:49, 30 October 2009 (UTC)[reply]
If you have access to a woolly mammoth for the dressing in a ruff, why can't you just use that access to also go in ahead of time to take a measurement? Dismas|(talk) 04:53, 30 October 2009 (UTC)[reply]
What? And spoil the surprise? Bielle (talk) 05:13, 30 October 2009 (UTC)[reply]
I was cheered up no end by thinking about woolly mammoths wearing ruffs! Readro (talk) 10:22, 30 October 2009 (UTC)[reply]
You will need Photoshop. Cuddlyable3 (talk) 17:37, 30 October 2009 (UTC)[reply]

The mammoth can be any size. The collar will have to be tailored to fit anyway.Cuddlyable3 (talk) 14:45, 31 October 2009 (UTC)[reply]

There are some museums which have life-size reconstructions of mammoths. Since there are real mammoths recovered in various states of decomposition from glaciers, these are likely to be fairly accurate, if recently constructed. I seem to recall a couple of such reconstructions outside the Cincinnati Museum of Natural History in early 1990, but the museum moved to new premises later that year, possibly discarding the mammoths, and these reconstructions are probably too old to be of use anyway.

Knee injury test

I was watching a documentary on football injuries. When testing for knee injuries, they did all the tests that I am used to seeing. Then, they did one that I don't understand. The person being tested took about one step forward with one foot. Keeping the back leg straight and bending the front leg, the person would apparently just shift his weight forward. Is this a standard test for a specific type of knee injury? If so, what is being tested? -- kainawā„¢ 03:19, 30 October 2009 (UTC)[reply]

Don't know specifically what the test is for, but that was a common exercise during physical therapy after my knee surgery. I know I did a lot of stuff related to the patella and making sure its accessory tendons and such were working to keep it properly centered, but I'm not sure if that specific exercise was related. ā€” Lomn 12:29, 30 October 2009 (UTC)[reply]
It may very well be rehab and not a test. I don't know why I didn't think of that. Thanks. -- kainawā„¢ 12:41, 30 October 2009 (UTC)[reply]
I'm not aware of this as a specific test. However the manoeuvre causes tension on the quadriceps muscle and the quadriceps tendon (and to a lesser degree on the posterior cruciate ligament). It could be used as a test of quad muscle/tendon integrity. Axl Ā¤ [Talk] 19:47, 30 October 2009 (UTC)[reply]
This is ringing a very faint bell for me. Slap a "citation needed" tag on this, but ISTR that they're looking at whether the patella shifts (left-right) as the leg moves. I've been looking through this rather impressive list of knee tests, but don't see what you're describing or what I'm remembering. Could be a place to look through, though. Matt Deres (talk) 23:58, 31 October 2009 (UTC)[reply]

Jumping on concrete versus sand

When we jump on concrete we get more hurt then when we jump on sand!can we explain this by third law of motion? My sir told me 2answer according 2the seceond law of motion but i answered it according 2the third law saying that since the particles of sand r very fine compared to that of concrete they r not able 2offer enough resultant force as concrete does.....so we feel more hurt?

Is it correct? ā€”Preceding unsigned comment added by 86.96.128.10 (talk) 10:06, 30 October 2009 (UTC)[reply]

Would it be the fineness of the sand particles, or the fact that they absorb more energy in the form of being displaced rather than posing a resilient barrier to the force and returning an equal an opposite force. It's like punching balloon (filled with air) or a punching bag. It's not that the air particles are smaller than whatever particle size composes the contents of a punching bag -- it's that the air moves away much more readily, allowing for greater dissipation of the imposed punching force. Unfortunately, though, I' well unaware of laws -- perhaps you can figure out to which law such a common sense approach would apply or be applied under. DRosenbach (Talk | Contribs) 12:21, 30 October 2009 (UTC)[reply]
You don't need any of Newton's laws of motion to explain this, although the second is the most applicable. The pain you feel is a result of the acceleration you're subject to when you land. Concrete doesn't give on impact, thus, your acceleration to a stop is very high (because the time for the change in velocity is very short). Per Newton's second law, that's a large force. However, if you jump onto loose sand (note the qualifier), it will shift around impact. That distributes your acceleration over a longer period of time, thus lowering its magnitude and the resultant force you feel. Note that jumping onto hard packed sand would be little different from hitting concrete. In any event, the difference is in how much whatever you impact moves. Newton's laws explain the peripheral factors (why time matters, why you hitting an object is like an object hitting you) but the core reason has nothing to do with them. ā€” Lomn 12:24, 30 October 2009 (UTC)[reply]


(ec)I think that's going into way more detail than is justified. Here is a simple explanation:
  • -- Newton's second law says that the force you feel (which is what hurts!) is the mass times the acceleration. The mass is your body mass - but what is the acceleration?
  • -- Equations of motion says that the final velocity squared (which is zero because you end up stationary) equals initial velocity squared (the speed you hit the ground squared) plus twice the acceleration you're going to feel - times (s-si)...which is the distance over which you slowed down.
So, we can simplify that second equation to:
where d is the distance over which you slowed down to a stop.
In other words - the force you feel when you hit the ground is your body mass times the square of the speed you hit the ground at divided by twice the stopping distance...and that's the key here. We all know that it hurts more to hit the ground while wearing a heavy backpack (because your mass is larger) and it hurts a lot more the faster you are falling - but what about that stopping distance? A large stopping distance reduces your acceleration compared to a short one. Since landing on solid, immovable concrete forces your feet to slow down in just the distance that the soles of your shoes & feet can be compressed - you only get (let's say) 5 millimeters of stopping distance. But if you land in soft sand, you slow down as the sand gets pushed out of the way. If you leave a 10cm deep footprint - then your stopping distance was 100mm - plus the 5mm for the compression of the soles of your feet and your shoes. That's 21 times more stopping distance in the soft sand. And that means 21 times less force applied to your feet - and considerably less pain!
Of course in reality, it's not just your feet. Hopefully you didn't land with your legs straight and knees locked! Presuming your knees were bent a little then the upper part of your body can slow down over the distance that your knees bend on impact. That softens the blow to your vital organs and brain considerably more than the soft sand does.
This same principle explains why cars are designed with 'crumple zones'. A totally stiff, rigid car would stop in almost zero distance if you drove it into a brick wall. But a well-designed car is designed to crush and crumple selected bits of metal - which allows the car to slow down over a greater distance - hence less acceleration and less force on the poor passengers. The airbag fulfills a similar purpose in giving your head more distance to slow down over than if it hit the steering wheel.
You can explain this in other ways - such as how the energy of the impact is absorbed - but this explanation actually boils down to more or less the same thing - and it's much easier to understand than trying to predict the cohesion of sand grains in loose sand versus sand locked within a matrix of cement.
SteveBaker (talk) 12:25, 30 October 2009 (UTC)[reply]
See also Jerk (physics). While your force and acceleration are the same in the two systems, the Jerk is the relevent value that changes, and that changes how damaging the force is. --Jayron32 12:29, 30 October 2009 (UTC)[reply]
I beg to differ. The momentum lost is the same in both systems but the acc(dec)elerations are different. Cuddlyable3 (talk) 15:32, 30 October 2009 (UTC)[reply]
But it's not the loss of momentum that hurts! You lose the same amount of momentum falling into a big soft pile of feathers as you do smacking into a block of concrete - but the results are most certainly not equivalent. SteveBaker (talk) 16:20, 30 October 2009 (UTC)[reply]
Very true. That is why airbags are made for cars, and cute little mini airbags are made for mini cars. Cuddlyable3 (talk) 17:32, 30 October 2009 (UTC)[reply]
This question is of great interest to people designing things likes sports and dance floors and playgrounds and there are various standards e.g. EN 14904 for sports floors. Basically we have evolved to have the equivalent of shock absorbers like in a car in our feet and joints. Ours seem to be tuned to running on grass or sand and not to jumping on rocks like goats. This is bad for us as modern streets and buildings typically have concrete pavements and concrete floors with a sheet of vinyl on top. Doing anything except walking on these is dangerous and they can be lethal for the elderly if they fall over. Dmcq (talk) 14:09, 30 October 2009 (UTC)[reply]

I'm amazed at the different ways responders formulate a "simple" answer, but not that SteveBaker works a car into the answer. It would only be Wikisensation if he treated a mechanical question without a car. Below is my explanation.
When you jump on concrete it is like the problem of what happens when an irresistable force meets an immovable object? The answer is that at the moment of contact one of them has to give way. Either the concrete has to shatter or (more likely) the foot meeting the concrete has to decelerate to a stop very quickly. Newton's 2nd Law Force = mass x acceleration shows that when acceleration has a large value (deceleration is just a negative acceleration) and mass is the mass of your foot, Force will be large enough to hurt your foot. The rest of your body also decelerates but if you were sensible and kept your knees bent a little, its deceleration is less and continues after the feet have stopped.
When you jump on sand the difference is not just that the sand has small particles, it is also that there is space or water between the particles, allowing them relative movement. Your foot hits the uppermost grains of sand but they cannot resist with as much force as the concrete. The grains are pushed downwards, impacting and rubbing against the layer of grains below. The top layer of grains is now effectively part of your foot and the friction below them is a small Force that gives a small deceleration. So your foot plus sand layer continues downwards a little slower. That impacts the next sand layer layer and so on, and your foot comes to a full stop only after having pushed some distance into the sand. That is a big difference from the near-instantaneous deceleration on concrete. Although the momentum (mass x velocity) that you lose is the same in both falls, in the fall on sand the Force of deceleration has been spread in time so its maximum value is less hurtful. Jump down on to a mattress and the deceleration is even less and the experience not hurtful at all. Cuddlyable3 (talk) 15:28, 30 October 2009 (UTC)[reply]

I have always wondered, since the law of conservation of energy is true, once your foot hits the concrete, where does the energy form acceleration go?Accdude92 (talk to me!) (sign) 15:31, 30 October 2009 (UTC)[reply]
Heat, sound (which soon dissipates into more heat) and breaking stuff (pulling apart the bonds between the atoms of the material). SteveBaker (talk) 16:17, 30 October 2009 (UTC)[reply]
To the OP, your Sir was right. In fact all 3 of Newton's Laws can be seen at work during the jumps. Energy is conserved but there is good work-producing energy and lazy good-for-nothing workshy energy. Before you jump down on anything, consider that your potential energy is a Non-renewable resource whose expenditure causes by the Second law of thermodynamics an increase in Entropy thereby hastening the Heat death of the universe.Cuddlyable3 (talk) 17:20, 30 October 2009 (UTC)[reply]
Interestingly enough the 5mm that SteveBaker used as a minimum is actually about the very minimum deformation a floor should have to avoid the main danger of an old person breaking their hip if they fall over. It's quite small. a full centimeter is much better but I think it shows the way our ancestors evolved to just about be safe on the plains in Africa without spending too much resource on over protection. Dmcq (talk) 23:09, 30 October 2009 (UTC)[reply]
Yeah, the Savannah Hypothesis is pretty much disproved these days, as much as such a thing can be. It's tempting to make up just-so stories to explain observations about humans, but we should be cautious: what's the minimum deformation a floor should have to avoid the main danger of an old chimp breaking their hip if they fall over? What about a cat? What about a kangaroo? Is there any reason to believe these are significantly different? 86.139.237.128 (talk) 00:13, 31 October 2009 (UTC)[reply]
We have evolved to run and walk whatever about what hypothesis or collection of them that is under. If we needed to jump around rocks like goats we'd have evolved much better shock absorbers in our legs and joints. Chimps don't have as much problem as us on the ground because they aren't as tall and their bones are far stronger than needed just for support. Humans however are built more for lightness and endurance than strength. Dmcq (talk) 16:26, 31 October 2009 (UTC)[reply]

The Verity Incident

What was the Verity incident? Theallwordslinkedtalkman (talk) 16:27, 30 October 2009 (UTC)[reply]

Googling the exact phrase "Verity incident" only turns up 10 hits, and they aren't all for the same event. And verity can mean a lot of different things. So it'd be hard to answer this question without the context in which the phrase was used. Red Act (talk) 16:49, 30 October 2009 (UTC)[reply]
It was an incident where a fugitive was arrested at gunpoint at a school, reported here and here. Cuddlyable3 (talk) 17:28, 30 October 2009 (UTC)[reply]
Do you by chance mean the Vela Incident? Googlemeister (talk) 18:22, 30 October 2009 (UTC)[reply]

If I were small enough could I observe an atom?

If I were as small as an atom could I watch its operation? Presuming there is a reality down there, would the atom be observable by a hypothetical supertiny person? Such a person could obviously not exist so this is a thought experiment. What would we see given the extreme speeds of the particles, their being point-like and I believe the particleā€™s speeds and position are undetermined until measured. Thanks for any thoughts (I suspect the question has no answer!) - Adrian Pingstone (talk) 18:47, 30 October 2009 (UTC)[reply]

The biggest problem is that the wavelength of light is larger than the atom, so you couldn't see anything, including yourself, if you were that small. Otherwise, it would be possible to observe the movement of nuclei, and probably observe how the electron "clouds" behave in bonds, individual atoms, ions, etc. but due to the speed of the electrons' movements, you would be unable to observe individual electrons. TheĀ SeekerĀ 4Ā Talk 19:13, 30 October 2009 (UTC)[reply]
Indeed, the problem is not that you are too big. The problem is that the resolution of light is inadequate. See "Microscopy", "Optical microscope" and "Electron microscope". The limit of resolution with light is about 200 nanometres. Axl Ā¤ [Talk] 20:00, 30 October 2009 (UTC)[reply]
These arguments are somewhat vague, as there exists near-field optics; also see the pages on near and far field, near-field scanning optical microscope and Near Field Communication (unfortunately, most of the articles are not very good, but you can check from journals that the idea works). The usual diffraction limits are calculated using the wave optics approximation to the Maxwell theory, an approximation that breaks up in the near-field case. Now on the original question: what should the hypothetical person consist of? Seeing means interaction between photons and the person's eyes, and the eyes' sensitivity depends on their composition.Ā ā€”Ā PtĀ (T) 20:31, 30 October 2009 (UTC)[reply]
One thing you could do is feel individual atoms, using the principles used by the atomic force microscope and the scanning tunneling microscope. Note that individual atoms are visible in the images in those articles. Red Act (talk) 20:45, 30 October 2009 (UTC)[reply]
Not only that but if you were really that small, brownian motion would beat the heck out of you! You'd better hope it's really cold! SteveBaker (talk) 22:16, 30 October 2009 (UTC)[reply]
I think it's actually the Heisenberg uncertainty principle more than anything else that limits the ability to observe an atom on a fine scale -- it says that the more precisely you know the position of an object, the less precisely you can know its velocity. This basically means that regardless of your size you can't know the fine details of an atom's motion. Looie496 (talk) 23:03, 30 October 2009 (UTC)[reply]
You can see as precisely as you want, but you have to choose beforehand, what exactly you are going to look at. You can measure the position of a photon exactly, but then you cannot know the momentum (p) of the same photon and, as E=hĪ½=pc, you neither know its frequency (its colour). Thus you have to choose at least one of bad spatial resolution and colorblindness. The same applies for any other pair of observables corresponding to noncommuting operators in quantum mechanics. However, if you decided to measure only colours, it would definitely be an interesting picture. At another time you may as well watch the spatial dynamics of the photons reaching your eyes. Quantum mechanics does not make everything blurry, it just bites when you want to learn too much at a time!Ā ā€”Ā PtĀ (T) 23:33, 30 October 2009 (UTC)[reply]
Actually, for such a small observer the whole concept of measurement changes as there the usual macroscopical decoherence does not happen anymore. The observer herself is a quantum object and we have no idea what a quantum consciousness in a notable superposition would sense. Note that the quantum mind is a different concept applied to try to explain the usual, macroscopical consciousness. It is all speculative indeed.Ā ā€”Ā PtĀ (T) 00:46, 31 October 2009 (UTC)[reply]
You could not "see" things if you were a human as small as an atom, because you would be far smaller than a single rod or cone light receptor in the retina, and you would be far smaller than a single nerve cell in the human visual cortex. For the scheme to work, you would have to hypothesize that you and all your organs were made of atoms many orders of magnitude smaller than the atoms you were observing. The converse would be, "If an atom were as big as a bus, could I watch its operation?" In that case you might have to hypothesize the giant atom having physical constants such as Planck's constant and the strong and weak nuclear forces and electrical constants many orders of magnitude different than in our universe. Edison (talk) 01:16, 31 October 2009 (UTC)[reply]
You need little teeny eyes/for reading little teeny print/like you need little teeny license plates for bees. --Trovatore (talk) 01:18, 31 October 2009 (UTC)[reply]

As the question is phrased, you obviously meant for us to take it in a COMPLETELY non-literal fashion. Being so small, nothing in your body would work -- what would you consist of if you were small enough to see an atom? The lumen of your digestive system would be microscopic, as would the lumina of your ureters and your blood vessels. Even cells would be too large to traverse your circulatory system -- I mean, the ramifications are so many, it's somewhat ridiculous to even begin listing them here. I therefore take your question to mean, "Should an atom be able to be visualized in real time, would the observer be able to perceive the various motions ascribed to, for example, the orbiting motion of the electrons." For my purposes here, and hopefully for your purposes as well, just as I am ignoring all the infinite problems associated with the viewing organism being too small to possibly be able to exist, so too am I ignoring all integral problems related to light microscopy and the possibility of viewing items less than the diameter of a wavelength of light -- for certainly, you would not be interested in a similar response pertaining to adjunct obstacles in the vieweing of an atom, such as, "well, you wouldn't be able to view an atom because, if you were so small, you'd be in the circus and wouldn't have time to look at atoms. Thus said, perhaps the editors involved above can focus on such a question -- if you're still interested and my assumptions were correct. DRosenbach (Talk | Contribs) 17:27, 1 November 2009 (UTC)[reply]

This is the OP writing: DRosenbach is exactly correct, I was indeed imagining that I was actually down there at the atomic level with eyes and other senses that can still function. I would love to receive any more ideas - Adrian Pingstone (talk) 22:23, 1 November 2009 (UTC)[reply]

Small populations sizes and genetic mutations

When a tribe is discovered in the jungle or a new group of people are found on a remote island, I assume they have a greater amount of genetic drift which presumably results in more frequent mutations. Are there any recorded cases of a small population being discovered and all the individuals involved having some kind of beneficial mutation? I'm no scientist so please try and answer in a way I can understand.Popcorn II (talk) 19:33, 30 October 2009 (UTC)[reply]

There won't be more mutations; a mutation happens in an individual, so it doesn't matter what's happening in the rest of the population. Genetic drift will result in them having different allele proportions from the population they split off from. In a small population it, combined with the founder effect, will result in reduced genetic diversity. That means it is quite likely that a small isolated population will have every individual having a particular allele, and that allele could easily be a beneficial one. (Few alleles that aren't shared among all humans are beneficial to everyone, otherwise they would become shared among all individuals, but a certain allele may be beneficial in their environment - that is now evolution works.) --Tango (talk) 20:00, 30 October 2009 (UTC)[reply]
There are clear examples of adaptive characteristics found in isolated populations (though I'm not sure that any have been mapped to a single causative mutation). One potential example is the Moken, whose children have remarkable underwater vision. Its not known whether their superior underwater vision is a genetic or learned trait since one can learn to accommodate one's visual focus underwater, but rarely to the extent commonly seen in Moken children. But its certainly beneficial, given they spend much of their time diving for food. Rockpocket 20:24, 30 October 2009 (UTC)[reply]
The Moken's underwater visual abilities are probably learned [8]--Gilisa (talk) 22:54, 31 October 2009 (UTC)[reply]
Ah yes, it turns out their follow up study showed that other children can adapt just as well with the correct training (PMIDĀ 16806388). What a shame, when I first read their paper back in 2003, it hinted at a beautiful example of an adaptive genetic characteristic. Rockpocket 00:53, 1 November 2009 (UTC)[reply]

Flu vs. other diseases

How deadly is influenza compared to other infectious diseases? In particular, what other parasites cause similar numbers of deaths in, say, the U.S.? Thanks. 66.65.140.116 (talk) ā€”Preceding undated comment added 20:05, 30 October 2009 (UTC).[reply]

According to the article Influenza vaccine, a report in 2008 cited that influenza accounted for about 41,000 deaths annually in the U.S. Worldwide figures can be found at Infectious disease#Mortality from infectious diseases, and influenza is counted there as part of a class called "lower respiratory infections", mixed in with things like pneumonia and stuff. So "flu-like" diseases are the largest cause of death from infectious agents worldwide, but I am not sure how this compares once you strip out the numbers for Influenza directly. Doing so would likely be impossible, since there are many non-influenza agents which cause nearly identical symptoms as influenza, and worldwide there is probably not the testing availible to seperate these. Diagnoses of "death from the flu" is probably made on a symptomatic basis, and as such, the best we can get on hard numbers would be "lower respiratory diseases". --Jayron32 20:18, 30 October 2009 (UTC)[reply]
If you're talking about mortality rate, flu is not terribly high on the list; it does get a high number of total deaths, though, in part because you can keep right on getting influenzas until one of them finally punches your ticket. Diseases with a very high rate of mortality include (in no particular order): HIV, Ebola, and untreated rabies (which, I think, was essentially at 100% until a small handful of people managed to pull through. Also, although their affects in humans may sometimes be quite similar, viruses and bacteria and generally not called parasites; that term is usually reserved for multicellular lifeforms. You may be interested in our article on List of causes of death by rate. Matt Deres (talk) 04:56, 1 November 2009 (UTC)[reply]

Tramadol expiry dates

Why do painkillers like tramadol carry an expiry date after which the instructions say you should not take them? Do they stop working? Or become dangerous? Or something else? Why? 86.166.155.90 (talk) 20:55, 30 October 2009 (UTC)[reply]

Generally the chemicals are not 100% stable and eventually break down, giving rise to byproducts which may or may not be harmful. Looie496 (talk) 21:04, 30 October 2009 (UTC)[reply]
Usually drugs just lose their effectiveness, but it is possible that the active ingredients, or one of the non-active ingredients that they use to make the bulk of the pill, will become potentially harmful. --Tango (talk) 09:32, 31 October 2009 (UTC)[reply]
About 2 yrs ago, I read an article on one of the common household NSAIDs and it mentioned that independent testing of the drug showed it was stable more than 2 years after the expiration date. The article suggested that expiration dates are as near as they are merely because the drug companies have only done testing for that long (e.g. 5 years). DRosenbach (Talk | Contribs) 00:02, 1 November 2009 (UTC)[reply]
A cynic might point out that keeping the period short sells more Advil. --Sean 13:41, 2 November 2009 (UTC)[reply]

October 31

Foreign accent syndrome and stroke recovery

I was curious about a case I read about a few years back, regarding a patient who had what, after searching here, I discovered to be foreign accent syndrome. It got me to thinking - if, after a stroke or other brain injury, in rare cases people can speak only in an accent, have rehabilitation workers ever experimented to see if a person who has lost their speech might recover it if taught to speak differently?

I'm aware there might be some flawed biology I'm not catching, but: If a person's neural patterns are such that they can't do the numerous things necessary to form speech, could there still be some mechanism that would let them produce it differently? could those with FAS speak that way because, in essence, they have "relearned to speak," in the same way a child learns to speak in his or her own unique voice? (Since every voice, even without an accent, sounds a bit different.)209.244.187.155 (talk) 00:32, 31 October 2009 (UTC)[reply]

Reading your question, I think you've missed that the people aren't really speaking in a foreign accent. It isn't that they've learnt to speak in a different accent, it's that brain damage affects how they pronounce some sounds in a consistent manner. People who hear them interpret this as them speaking in a foreign accent. For example, someone who previously had a rhotic accent might suffer brain damage that affected their ability to pronounce the 'r' in words like 'Arthur'. Their friends and family, who also speak with rhotic accents, perceive this as them speaking a non-rhotic accent (like many British accents). 86.139.237.128 (talk) 02:32, 31 October 2009 (UTC)[reply]
Okay, you're right, that's what I was getting confused by; thanks.4.68.248.130 (talk) 09:15, 31 October 2009 (UTC)[reply]
Stroke symptoms can be very strange, and it's hard to predict what might or might not be possible. There are, for example, cases of people who have lost the ability to talk but are still able to sing. Looie496 (talk) 04:27, 31 October 2009 (UTC)[reply]


I heard from the Media about one Czech turist who had traumatic brain injury after falling from his bike in London. Then, when he woke up in the British hospital he start speaking in fluent English with British accent, something he certainly couldn't done before. So, it seems like the brain could be very flexiable sometimes (but not always). For instance, it seem like the visual cortex of people who lost their sight is being used extensively for the sense of touch -this ability of the brain to gain new functions or to rewire itself isa part of its neural plasticity. But what you asked is more complicated, as we are talking about something more complicated than sensing-language is learning dependent high function and it can't appear out of the blue. So it seem much more similar n to one regression to his/hersecond language, that happened many times during degenerative diseases such as Parkino's disease. That is, one major function is lost, so the brain must use the more preserved areas and the previously dominant brain areas can't longer overpower/inhibit the areas that contain these secondary abilities.--Gilisa (talk) 17:39, 31 October 2009 (UTC)[reply]
I am glad he survived. It is sad to see a Czech bounce. Cuddlyable3 (talk) 12:43, 1 November 2009 (UTC)[reply]

Increasing the size of the human penis?

Is there any medications available that actually work to permanently increase the size of the penis? ā€”Ā Preceding unsigned comment added by 84.68.36.19 (talk ā€¢ contribs)

We have an article on that: Penis enlargement. Red Act (talk) 01:35, 31 October 2009 (UTC)[reply]
No. [9] ā€”Preceding unsigned comment added by 79.75.106.210 (talk) 01:50, 31 October 2009 (UTC)[reply]
You wouldn't enjoy permanent enlargement this way though there is a popular temporary way. Cuddlyable3 (talk) 14:40, 31 October 2009 (UTC)[reply]


I always get e-mails that offer me to buy enlargement kit-all I have to do is to give my address and c. card number and a simple kit will be sent to my post office box. I never tried it, but maybe you want me to give you connection details?--Gilisa (talk) 18:12, 31 October 2009 (UTC)[reply]

With all due respect, I hope you're kidding Gilasa. Let me put it this way: if the information is in your spam box, the information fails being trustworthy as soon as your credit card information changes the outcome. Magog the Ogre (talk) 20:12, 1 November 2009 (UTC)[reply]

Nifty Propulsion Device

Someone pointed this thing out to me [10] and I don't know enough physics to determine whether it makes sense. Can someone who knows more about it give a ruling? Black Carrot (talk) 04:45, 31 October 2009 (UTC)[reply]

This is the EmDrive. As a reactionless drive, it blatantly violates the conservation of momentum and cannot work as claimed. After the magnetron is turned on, the microwaves aren't going to do anything except bounce around and get absorbed with absolutely no effect on the cavity's total momentum. The energy will then be radiated as blackbody photons, though asymmetries in the device's geometry may provide a miniscule force on the "drive". In other words, the "drive" will function much better if the end facing the magnetron is removed, because the photons will then serve as a tiny reaction mass. --Bowlhover (talk) 06:36, 31 October 2009 (UTC)[reply]
Someone who knows more about it has given a (rather damning) ruling, and you can see it here: [11]. The basic error Shawyer makes is to assume the impulse imparted on a wall during a particle-wall collision is in an incorrect direction. You can see this yourself by looking at figure 2.4 of [12] (an earlier version of the same document). In the "updated" version, Shawyer removes the section which makes his error obvious. Someone42 (talk) 07:00, 31 October 2009 (UTC)[reply]
Roger Shawyer claims[13] to have calculated and measured on a demonstrator engine 16mN thrust from an input power of 850W. Dr. Costella[14] accuses Roger Shawyer of being a charlatan who defrauded a government agency (the UK Department of Trade and Industry). I smell a libel suit where lawyers are bound to earn. Cuddlyable3 (talk) 14:35, 31 October 2009 (UTC)[reply]

The only real test will be when someone actually takes his device up into orbit and it moves.When is this going to happen?80.0.98.26 (talk) 11:14, 1 November 2009 (UTC)Trevor Loughlin[reply]

Hopefully never, because that would cost a lot of money and I would hope that the people who control such quantities of money would do a bit of investigation before giving it away. Seriously, the guy is a complete idiot (or a con man). He doesn't even claim some hitherto undiscovered law of physics, he claims that his drive works in the framework of standard electrodynamics. But it's a theorem that momentum is conserved in electrodynamics (Noether's theorem), so he's wrong and that's the end of it. The most valuable thing about this fiasco is that it's a lesson in just how far New Scientist's journalistic standards have fallen. It's barely above the level of a supermarket tabloid at this point. The problem isn't so much that it's a terrible magazineā€”there are lots of thoseā€”but that people still trust it as a source of physics news. It's disturbing how often it gets used as a reference in Wikipedia articles. Greg Egan wrote a plea to save New Scientist in the wake of this story, but I think it's too late for that. I would instead plead with people to cancel their subscriptions. -- BenRG (talk) 13:23, 1 November 2009 (UTC)[reply]

Dry friction

What actually is dry friction? Can i call viscosity as fluid friction and vice-versaĀ ? ā€”Preceding unsigned comment added by 122.168.213.193 (talk) 11:17, 31 October 2009 (UTC)[reply]

Not really. Dry friction is the lateral friction between touching surfaces of solids, and may be classified as either static or dynamic depending on whether the surfaces are stationary or moving relative to one another. Viscosity is the resistance of a fluid to flow, and whilst the name "fluid friction" sounds appropriate, it's ambiguous as the term is already taken to be the friction between two solid surfaces separated by a fluid, or sometimes the friction between layers of a fluid with relative motion to each other. See this.--Leon (talk) 13:55, 31 October 2009 (UTC)[reply]

Area dependency of friction

It is stated that frictional force is independent of area of contact then why we fell it harder to ride a bicycle with flat tire (air less) ā€”Preceding unsigned comment added by 122.168.213.193 (talk) 11:26, 31 October 2009 (UTC)[reply]

First let me point out (before Steve Baker starts jumping up and down) that that description of the friction force as independent of the area is intended for non-sticking flat hard surfaces. Tires don't meet those criteria. More importantly, though, you are confusing rolling friction (on the bike's tires) with sliding friction (which is the kind of friction this description is intended for). Dauto (talk) 11:34, 31 October 2009 (UTC)[reply]
Sliding is not a factor. Squashing and unsquashing the rubber of a flat tyre consumes (i.e. converts to heat) energy, see Rolling resistance, that must be provided by the rider. Cuddlyable3 (talk) 14:11, 31 October 2009 (UTC)[reply]

A noise meter to measure outdoor road noise pollution in the UK?

Noise or sound meters are less expensive than I thought, and buying is cheaper than hiring. I want to measure outdoor road noise pollution, in a way that complies with UK standards for doing so - whatever they are. I think this requires using an "A" digital frequency filter, which seems available on many meters. But rather than looking at a fluctuating meter and mentally trying to average the reading, are there any meters that will do this automatically - that will tell you what the average reading was over a minute say? Since I will avoid rainy situations a waterproof meter is not necessary. Thanks 78.144.206.114 (talk) 11:37, 31 October 2009 (UTC)[reply]

It sounds (pun) that you want to measure A-weighted long-term average sound level as defined in BS 7445 / ISO 1996. Look for a noise meter that has an analog or digital logging output that you can connect to a PC. If it is analog then you need an A-to-D converter. THe PC can store the data, calculate average(s) and show them on a spreadsheet. A common mistake made by professionals who should know better is to take an average of dB(A) measurements. This is wrong because dB (decibel) is a logarithm of a power measurement. The correct way is to convert from the dB(A) values to power(A) values, take their average, then convert back to dB(A). For the legal background in the UK see [15] the Noise and Statutory Nuisance Act 1993 and the EU Environmental Noise Directive. Cuddlyable3 (talk) 14:05, 31 October 2009 (UTC)[reply]

Thanks, although where I want to take the noise readings will be a very long way from my desktop computer. Are there any that can do the "averaging" themselves? Thanks. 78.151.126.29 (talk) 21:35, 31 October 2009 (UTC)[reply]

Consider using a portable Laptop or Netbook. Battery time is a limitation. Cuddlyable3 (talk) 22:05, 31 October 2009 (UTC)[reply]

Cicely of Oxford Royal Physician

I read somewhere that Cicely of Oxford was Court Physician to either Richard II or England or Edward III of English. Is there any way to confirm this? ā€”Preceding unsigned comment added by 76.200.166.39 (talk) 11:51, 31 October 2009 (UTC)[reply]

Both [16] and [17] claim that she was Court Physician to Edward III, although neither look particularly reliable. The whole claim looks rather doubtful, as I cannot find any other references to Cecily or Cicely of Oxford, and historical works which would have been expected to mention a fact so interesting as a fourteenth-century female physician (e.g. this survey of royal physicians in the reign of Edward II and immediately after) don't have anything to say about it. Warofdreams talk 23:06, 31 October 2009 (UTC)[reply]
P. C. Doherty is a pretty bad writer of detective stories, but he has strong academic credentials as a 14th century history, so if he claims Cicely of Oxford was physician to Edward III, he probably has a good basis for saying so. Looie496 (talk) 02:49, 1 November 2009 (UTC)[reply]

Epidermal growth factor

Hi I'm a little confused about how epidermal growth factor (EGF) is produced in the human body. Are there cells that produce it? The article on EGF says 'sources' are macrophages, platelets, urine, plasma and milk. What does this mean? Do macrophages and platelets produce EGF? I can't imagine urine/milk producing anything, or does it mean ingesting urine/milk would provide a person with a source of EGF? Any clarification would be great, thanks RichYPE (talk) 14:20, 31 October 2009 (UTC)[reply]

I think you have to go to the cited source of the information which is Cotran, Ramzi S.; Kumar, Vinay; Fausto, Nelson; Nelso Fausto; Robbins, Stanley L.; Abbas, Abul K. (2005). Robbins and Cotran pathologic basis of disease. St. Louis, Mo: Elsevier Saunders. ISBN 0-7216-0187-1.Cuddlyable3 (talk) 21:33, 31 October 2009 (UTC)[reply]
Epidermal growth factor is a protein, and as such is produced by protein synthesis, which occurs in cells. The megakaryocytes that give rise to platelets, synthesize EGF (see this paper, as do macrophages. When EGF is found outside of cells, it is because it has been secreted from the cells that produced it. --NorwegianBlueĀ talk 11:12, 1 November 2009 (UTC)[reply]

Hydrocephalus

good evening, I just want to ask about hydrocephalus.. i am a Physiotherapist, and i have a patient who have one, my main concern is that she keeps on crying while the therapy is on going, and im afraid if this is bad or dangerous to her, sometimes she turns cyanotic.. hope you can help me and also I want to know what technique can i improve her trunk control for she will be able to sit and walk independently... waiting for your response": Erlinda M. Mendoza ā€”Preceding unsigned comment added by 120.28.71.159 (talk) 15:46, 31 October 2009 (UTC) ā€”Preceding unsigned comment added by 86.4.186.107 (talk) [reply]

Erlinda, I'm sorry but we don't give medical (or physiotherapy) advice on the Reference Desk. Receiving medical advice from strangers for the treatment of a third person is very risky, I'm sure you can understand this. If you are a chartered or qualified physiotherapist you must surely have professional colleague or seniors who can give you advice. Of course we have an article on hydrocephalus, reading this might be helpful. Caesar's Daddy (talk) 16:04, 31 October 2009 (UTC)[reply]

Sounds of a Teaspoon in a Mug of Milk?

Take a mug of milk (or milky tea, or milky coffee). Heat it. Take a teaspoon, and tap it on the base of the mug (but from inside the mug, like you're stirring it.) Listen to the sounds, specifically the pitch of the sound. WHAT CAUSES THIS? ASCII image related.

Ā Ā Ā |
#Ā Ā |Ā Ā Ā #
#--|---# Teaspoon in a mug with hot milk,
#Ā Ā |Ā Ā Ā # move it up and down, tapping mug base,
#Ā Ā |Ā Ā Ā # what the hell are the sounds like that for?
#Ā (Ā )Ā Ā #
########

No answers from anyone what has not tried this please. Remember Civility (talk) 19:01, 31 October 2009 (UTC)[reply]

Before putting in the milk the mug gave out a tinkle, effectively acting as a bell. As the milk went in the resonant frequency went up because of the smaller part of the mug that can resonate freely, but changed progressively from a tinkle to a dampened knock sound. Then I drank the milk yum-yum. Cuddlyable3 (talk) 21:28, 31 October 2009 (UTC)[reply]
That's "Original Research", I'm not sure that's allowedĀ ;-)) Richard Avery (talk) 22:45, 31 October 2009 (UTC)[reply]
YOU DIDN'T TRY IT DID YOU? :p Take a full mug of milk. Tap. Listen to that sound. Tap again (not adding or removing any fluid). Listen to that sound. tap - tap - tap - tap - tap - tap - tap - tap, listen to the sounds of the taps. No liquid added or removed. Remember Civility (talk) 22:49, 31 October 2009 (UTC)[reply]
I tried it with a heated mug of water (my girlfriend drinks milk, but there's none in the fridge at the moment), and it sounded quite unremarkable. There are a lot of variables here that will affect what is heard with this experiment -- the thickness, shape and material of the mug, how full the mug is, the acoustic properties of the surface the mug is placed on, possibly the thermal properties of the liquid, mug, and surface, since the liquid is heated, etc. Could you please record the sound and upload it, so we can hear whatever it is you're hearing? Decent laptops nowadays have built-in microphones (even if you can't necessarily see where it is), it's easy to record using the Sound Recorder that comes with Windows, and I think it's fairly straight-forward to upload a file using the "Upload file" link under the "toolbox" section of this page. Red Act (talk) 01:34, 1 November 2009 (UTC)[reply]
As someone who has heard this noise, I can tell you that what causes this is the same thing that causes any sound. When the spoon hits the mug, the spoon and mug vibrate. The milk in the mug affects the vibrations of the mug and the spoon, and what you hear is a combination of all the vibrations. See sound ā€”Akrabbimtalk 01:04, 1 November 2009 (UTC)[reply]
Most of the respondents don't seem to have noticed the phenomenon that I think Remember Civility is referring to: correct me if I'm wrong, RC, but I believe it is that when the mug is tapped, the pitch of successive taps progressively rises for a time. I've noticed and wondered about this when stirring hot tea or coffee: at first I assumed it was due to the rising temperature of the mug (due to heat transfer from the beverage) altering its acoustical properties, but it also seem to happen if one waits long enough before stirring for the beverage and mug temperatures to have stabilised. An alternative conjecture is that tapping (or stirring) releases dissolved gasses in the beverage, altering its density and therefore acoustical properties. I too would really like to know the correct answer to this. 87.81.230.195 (talk) 06:35, 1 November 2009 (UTC)[reply]
I haven't tried this (not a tea drinker), but I've certainly noticed something which might be similar. If you fill a glass with hot water from the faucet, and drop some silverware in, it rattles with a sound which I would describe as a knock or a tap. But a few minutes later, if you jiggle the silverware around, the sound is much more tinkly. (And I notice this phenomenon all the time, when I'm soaking the glass and the silverware prior to washing them.)
There's something else I notice. When I first run the hot water into the glass from the faucet, it's cloudy. A few minutes later, when the sound has changed, the appearance has changed, too -- now the water is clear.
You can probably see where I'm going with this, and it's the same suggestion 87.81.230.195 made already. Hot water fresh out of the tap contains a lot of dissolved gases. The dissolved gas seems to quite significantly muffle or deaden sound. It takes a few minutes for the gas to come out of solution, and when it does, both the appearance and the sounds are much clearer. ā€”Steve Summit (talk) 14:35, 1 November 2009 (UTC)[reply]
I'm fairly certain the cloudiness is due to temporary hardness (not dissimilar to the limescale in the kettle). Whether this has an effect on the noise, I'm not sure, but it's possible. - Jarry1250Ā [Humorous? Discuss.] 15:35, 1 November 2009 (UTC)[reply]
editors who've noticed the rising pitch are correct. I didn't want to mention it for fear of "planting" the thought. I've done a bit more search and it is mentioned in an old [i]New Scientist[/i], see this link, and then, 14 years later, they have another, fuller, bit of text. Note that stirring can "reset" the sound back to a lower pitch. Remember Civility (talk) 15:33, 1 November 2009 (UTC)[reply]
I just did some original research regarding this question, in a very literal sense of the phrase: I have now tried this three times with pure boiling hot water, and three times with boiling hot water with cocoa powder in it. All three times pure water was used, there was no discernable rise in pitch. All three times cocoa powder was used, there was a rise in pitch.
The third time each way was done simultaneously, and performed identically except for the cocoa powder: I brought identical amounts of water in identical mugs to a boil at the same time in a microwave oven. I moved the two mugs simultaneously from the microwave to adjacent tiles on the same countertop. I poured some cocoa powder into one of them. I stirred both mugs simultaneously with identical teaspoons briefly, and then simultaneously held the two spoons in a bottom corner of their respective mugs briefly, to essentially eliminate the swirling motion of the liquid in each. I then repeatedly tapped the bottom centers of the mugs with the spoons in alternating order. The pitch in the pure water, to the best of my hearing ability, stayed constant. The pitch in the cocoa started off considerably lower than in the pure water, and ended up if anything slightly higher. In neither case did bubbles come to the top of the liquid during the tapping process.
It certainly appears to be quite important that there be a solute in the water. However, it isn't necessary for the solute to be milk, as the OP suggests, since the cocoa powder I used does not contain any milk. The cocoa powder I used was Ghirardelli Sweet Ground Chocolate and Cocoa, which contains sugar, cocoa (processed with alkali), unsweetened chocolate, soy lecithin and vanilla.
The "tiny bubbles" hypothesis put forward by Cecil Adams[18] seems dubious to me, as I could not see any bubbles reaching the surface of the liquids during the tapping process. And the "changing vortex" hypothesis[19] doesn't make sense, either, because the liquid was essentially stationary in both mugs during my final experiment. Red Act (talk) 17:57, 1 November 2009 (UTC)[reply]
If stirring the liquid really does "reset" it - then it has to be something to do with the vortices (or lack thereof). Bubbles might be knocked out of solution by stirring - but it's hard to imagine that more could be added - hence stirring couldn't reset things if bubbles were the cause. Stirring won't reset the temperature - so the effect of gradual cooling can't be the cause either. The idea that the liquid has to be moving in some specific manner probably accounts for some of what's going on. Sound waves will get dragged around with the fluid - so you could imagine different resonant pathways in rotating fluid versus stationary. Using a curved spoon to do the tapping will certainly start up some motion of the liquid - and that might easily be "reset" by stirring. Changing the ingredients of the liquid would change viscosity and perhaps the speed of sound - so it should be no surprise that this is a determining factor. SteveBaker (talk) 22:00, 1 November 2009 (UTC)[reply]
How about this for a hypothesis: Suppose the liquid is spinning (eg after stirring) in a smooth, circular path with more or less laminar flow. In stationary liquid, the sound from a tap made in the center would travel radially outwards until it hit the side of the cup - but in a spinning liquid, it has to travel around in a spiral. This changes the distance it has to travel thereby altering the resonant frequency - and also alters the angle at which the sound waves hit the side of the cup changing refractive patterns of the sound through to the air outside. Either of those things could change the pitch that you hear. However, because you're using a complicated curved spoon to do the tapping, each movement of the spoon causes turbulant eddies to spin off from it. Progressive taps cause more and more turbulance - and after enough taps, the spinning of the liquid is broken up into much more random motion at small scales and very little laminar flow - resulting in approximately equal travel times for the sound. Stirring the liquid restores the large-scale laminar flow and allows the experiment to be repeated.
Suggested experiments:
  1. Try tapping with a thin, cylindrical object (a glass rod would be perfect) - trying to keep the rod in the center of the cup. This ought not to disturb the flow of the liquid so much - so (if my hypothesis is correct) it should take many more taps to mess up the flow rate and get the pitch of the sound to rise.
  2. Try messing up the flow of the liquid with random motions of the spoon instead of smoothly stirring it. If the hypothesis is correct, this should immediately "reset" you to the highest sound frequency instead of the lowest.
SteveBaker (talk) 22:11, 1 November 2009 (UTC)[reply]

November 1

Numbering of Magicicada broods

Why are the Magicicada broods numbered the way they are? For Brood I (for 17-year cicadas), the year number of emergence leaves a remainder of 6. Why not 1, or 0? ā€”Preceding unsigned comment added by 75.37.158.42 (talk) 04:16, 1 November 2009 (UTC)[reply]

They are probably numbered after the first year someone started numbering them. But the numbers are clearly intended to be fairly arbitary because they numbered the 13 year cicadas with the next available numbers after the 17 year variety. I don't think there is anything particular to read into the choice of numbers. SteveBaker (talk) 21:52, 1 November 2009 (UTC)[reply]

What specific type of foam is this? (Think tool/bike grips)

Can't find a definitive answer on this anywhere. There are two specific materials I can't identify that I see used in handles/grips of all kinds; ex: tools, fishing rods, bike handlebars, etc..

The first one is in tube form, with an 1/4" wall. It feels squishy and soft (firmer and deforms less than visco-elastic foam) and has a somewhat rubbery feel. The inside of the tube is smooth and shiny like neoprene you'd fnd in a wetsuit or gasket, and the outside has a matte, smooth texture that I assume makes this open cell foam. The second material is similar, except the cell size is much larger, and thus the texture is more 'pebbly' as. It is in a sheet 1/4" thick.

Other than that (admittedly) poor description, all I can add is: the material isn't 'plasticky' feeling like extruded polyurethane tubing for pipe insulation, nor is it low-density like polyurethane foam for upholstery. It's also not so dense that it'd hurt you if someone threw a chunk at you.

Looking around, the likely suspects I've found across were EVA and a PVC variant (may be a trade name) called NPVC. Neoprene also, but I rarely see it in the thicknesses used in these products, or with any texture. So, does anyone know exactly what I'm talking about? Please don't say foam rubber. That much is obviousĀ :) Thanks in advance.

KA - 97.82.253.69 (talk) 04:28, 1 November 2009 (UTC)[reply]

NPVC stands for Nitrile PVC. It is pretty much the standard for foam grips. There are many variants of it because it is easy to adjust the formula when producing the grips. -- kainawā„¢ 04:39, 1 November 2009 (UTC)[reply]

I couldn't have asked for a better answer. Many thanks. 97.82.253.69 (talk) 09:29, 1 November 2009 (UTC)[reply]

Power vs frequency

Why does the power/torque of an Internal Combustion engine depend on its RPM? ā€”Preceding unsigned comment added by 116.90.224.116 (talk) 07:00, 1 November 2009 (UTC)[reply]

You can answer that loads of ways but basically RPM gives the rate at which fuel is burned (RPMxnumber of cylindersxamount of fuel per cylinder) or alternatively the number of explosive pushes by the cylinders which the engine harnesses. So at low RPM power is proportional to RPM but at high RPM the efficiency falls away for several reasons. If you give the approx level of the question it is easier to pitch the answer. --BozMo talk 09:55, 1 November 2009 (UTC)[reply]
For the same reason that you have to breathe faster to run faster. Cuddlyable3 (talk) 12:34, 1 November 2009 (UTC)[reply]
I'm going to do some simplifying here, but broadly what I'm saying is correct. Torque depends on RPM because the engine isn't equally efficient throughout its rev range. At low revs engines typically do not fill and empty the cylinders efficiently. At high revs it's often the case that the valve openings are too small to allow the combustion gases in and out of the cylinder fully. As a result, maximum torque is often somewhere in the middle of the engine's rev range. Power is given by torque multiplied by revs, so, if the torque was constant over the rev range, power would linearly increase with revs. Since torque drops off at high revs, we find that power will eventually do so as well, but because of the rev multiplier, it's always higher up in the rev range. --Phil Holmes (talk) 18:58, 1 November 2009 (UTC)[reply]

lightening shock

if electricity is been harnessed from lightening why does the rain water that flows from the same place conduct? what s the mechanism behind that? ā€”Preceding unsigned comment added by Srividhyaathreya (talk ā€¢ contribs) 08:02, 1 November 2009 (UTC)[reply]

Each little drop of water carries a small static charge. Electricity is not being conducted in the normal sense of electrons flowing in a medium, but though the bulk travel of charged drops. The air only conducts when a high voltages starts to cause a breakdown. Graeme Bartlett (talk) 10:59, 1 November 2009 (UTC)[reply]

career

i m an electrical engineer from India. i wish to enter into the field of space science. can i pursue an m.e or m.s. in aerospace engineering. if not what is the apt course to become an astronaut or a space scientist? ā€”Preceding unsigned comment added by Srividhyaathreya (talk ā€¢ contribs) 08:05, 1 November 2009 (UTC)[reply]

I would expect that you may be employed as a support person, however to be an astronaut, you should qualify as an aircraft pilot first. As a space scientist, you had better get a PhD in physics. Graeme Bartlett (talk) 10:56, 1 November 2009 (UTC)[reply]
You don't need to be a pilot to be an astronaut. Pilots of spacecraft will generally be pilots, but there are scientists and engineers that become astronauts too. I would expect a PhD is required to do scientific research in space. --Tango (talk) 19:43, 1 November 2009 (UTC)[reply]
See Astronaut#Training for NASAs requirements. Other space agencies will be similar. --Tango (talk) 19:45, 1 November 2009 (UTC)[reply]
Right, there are lots of different researchers in space, not just physicists, and definitely not just pilots... it is not the 1960s anymore. --Mr.98 (talk) 20:45, 1 November 2009 (UTC)[reply]

Inflammatory result of depolarization theory regarding migraines

That title could've been better. But! I've been reading up on your excellent migraine article, and it lists, among other things, depolarization and serotonin levels as resulting in pain. It is the first I am interested in, because browsing through depolarization and action potential, I am hard pressed to understand exactly what it is that causes an inflammatory agent to be released when a depression occurs. I am quite the noob on this field, and so I feel I may have missed something. As I have understood this matter, a depolarization essentially inhibits to a great extent the cells' ability to communicate, suffering from reduced or nonexistant permeability. Under any circumstance, your help is much appreciated! 91.149.2.165 (talk) 10:40, 1 November 2009 (UTC)[reply]

Studies in psychoneuroimmunology have shown that during depression proinflammatory neuropeptides (SP) and cytokines (e.g., IL 1 and IL 2) are released as a result of over activation of the sympathetic nervous system and the HPA axis->leading to acute phase response and to the release of Acute-phase proteins. This is only part of the phsyiological effects of different negative emotional situations.--Gilisa (talk) 11:46, 1 November 2009 (UTC)[reply]
I am terribly sorry for having used the word "depression", when I in fact meant cortical spreading depression! I understand your answer thus has less validity.Ā :) 91.149.2.165 (talk) 12:19, 1 November 2009 (UTC)[reply]
Actually it does have connection after all as cortical spreading depression may lead to the release of SP, but I don't familiar with works on this mechanism and neuroimmunology is not my expertise. I addressed your question to Dr Dima, he may know the answer.--Gilisa (talk) 13:03, 1 November 2009 (UTC)[reply]
Absolutely marvellous! Thank you for taking your time on this issue, I certainly hope for a good answer =) 91.149.2.165 (talk) 13:54, 1 November 2009 (UTC)[reply]
You welcome! I'm hoping for a good answer myself.--Gilisa (talk) 14:24, 1 November 2009 (UTC)[reply]
I will start from the end. When membrane of a neuron is depolarized by injecting electric current into the neuron, such depolarization leads to increased spiking probability. However, membrane depolarization in general does not imply increased spiking (and membrane hyperpolarization in general does not imply depression). When membrane is depolarized due to change in ion concentrations in the cell and around it, or due to change in conductances of particular type(s) of ion channels, such depolarization may be associated with either increased or decreased spiking probability. There is no contradiction, therefore, between depolarization and depression in the cortical spreading depression in a migraine. That being said, I do not know what exact changes in channel conductances and in the ion balance occur in the neurons in the areas affected by the spreading depression. I never worked on migraine or anything closely related. What I do know is that the depolarization is thought to be not produced by serotonin directly. According to the theory I am somewhat familiar with, mutations in a certain subunit of the voltage-gated P/Q-type calcium channel, or possibly other "channelopathies" in the brain stem, result in the over-active vascular response to the increased activity in certain areas of the cortex (see e.g. this article). Change in blood supply affects the activity in the cortex, which affects the blood supply even more, closing the positive feedback loop. Serotonin, it seems, mostly acts on the blood vessels and not on the cortex neurons in this case. However, with serotonin you never know, as most of its effects are modulatory rather than direct; and I am certainly not a migraine expert. I hope this helps in any way. --Dr Dima (talk) 17:46, 3 November 2009 (UTC)[reply]

subsequent natural GMO

How likely or what is the probability that a GMO such as corn with a GM to produce an insecticide or a cow designed to give milk with antibodies could undergo subsequent natural GM from a virus or pollutants or electromagnetic radiation resulting in an increase in production of insecticide or antibody which might render the GM product harmful? Biggerbannana (talk) 11:29, 1 November 2009 (UTC)[reply]

About the same as ordinary corn or an ordinary cow. We can't predict anything else. Cuddlyable3 (talk) 12:30, 1 November 2009 (UTC)[reply]
Perhaps then it depends upon where something grows but I can't find a map anywhere of areas with high numbers of trees that have large cancerous type growths on their trunks although they appear to be in areas near where phosphorus was discovered and is mined in South Florida and in other areas most likely to have phosphorous deposits which has a lot of radioactivity. Biggerbannana (talk) 14:50, 1 November 2009 (UTC)[reply]
In "captivity", these products' reproductive cycles are fairly closely tracked - it's unlikely that anything untoward would happen. The issues arise if they escape into the wild. In that case, those transgenic genes would continue to work and to be selected for (or against) and be possible targets for mutation. However, if the genes don't confer any particular benefit to the organism in the wild (for example, a sheep that produces human insulin in her milk) - and since the organism has to expend energy to produce these extra things - the odds are good that they'd be selected out after enough generations. If they DO offer benefits (resistance to certain common insects) - then they might be strongly selected for - and spread out into the wild population. There is evidence that this has happened already with Starlink corn. However, evolution is only going to operate in ways that strictly benefit the organism - which means that there has to be a balance between the cost to the organism to produce whatever this effect is - and the benefits. The fact that wild versions of the organism didn't already evolve this kind of protection suggests that the cost may have been higher than the benefits - but it's really tough to know. Of course it's much less likely that it would be a virus, pollutants or radiation causing the gene to mutate (a relatively rare event) so much as it's likely that mixtures of the transgenic gene with other (relatively rare) genes already out there in the wild population would increase the effectiveness of the transgenic gene in some way (eg by feeding it's chemical pathway with more inputs). So the odds aren't zero that the release of transgenic organisms might cause gradual evolution of novel new features. SteveBaker (talk) 15:25, 1 November 2009 (UTC)[reply]
Sounds like what you are saying is that because the two conductor extension cord I found at a local South Korean flea market had blades made of pot metal and 26 gauge copper wires covered by thick insulation attached only by the pressure of the molded plug holding the blades and wires together is okay even though they violate the electrical standard because they will melt quickly and act like a fuse despite the unlikely possibility of not melting quickly and instead heating up slowly resulting in a fire. Biggerbannana (talk) 16:44, 1 November 2009 (UTC)[reply]
That is one of the worst rationalizations I've read here - to the point that it is very difficult to assume that you are actually asking a question and not just ranting about GM products based on little (or no) actual knowledge of GM. He stated that evolution works towards the benefit of the organism. If a genetic modification does not benefit the organism, it will not tend to spread. Try - though I know it is very difficult - to relate that to an organism, not an extension cord. -- kainawā„¢ 20:10, 1 November 2009 (UTC)[reply]
The reference being that anyone can produce inferior goods or follow an insufficient regimen to make money without dependence upon repeat buyers. Biggerbannana (talk) 04:37, 2 November 2009 (UTC)[reply]
(It's of note, of course, that "benefit of the organism" is not the same thing as "benefit to people." Mutations in viruses that are good for the organism are often quite bad for humans!) --Mr.98 (talk) 22:49, 1 November 2009 (UTC)[reply]
I will also add that although what SteveBaker says is correct, it's possible that increased production can be harmful to other creatures (such as humans who eat the cow or corn), although usually indirectly e.g. massively increased production of antibiotics creates promotes resistance in those infectious agents, thereby making the disease harder to cure in humans. ~ Amory (u ā€¢ t ā€¢ c) 21:19, 1 November 2009 (UTC)[reply]
It may be worth remembering that many plants that we eat already have a number of compounds which are clearly harmful to humans in sufficient quantities. E.g. solanine in potatoes. A far greater concern then any potential (direct) harm to humans is the ecological effects of transgenes. In particular, if the transgene is beneficial to the plant but harmful to something else (e.g. insects or other plants) if the transgene spreads as SB mentioned and spreads widely enough this may have a negative effect on the ecological balance. A key issue is whether and what other wild species the transgene can spread to, as most plants cultivated for food don't survive particularly well in the wild because they're so screwed up, the addition of whatever transgenes is probably not enough to make them into weeds. Note in case it isn't obvious, this doesn't usually doesn't require any mutation. A terminator gene would hopefully nearly eliminate the spread. All this is something that has received plenty of research, so you could easily find many references if you are interested Nil Einne (talk) 22:55, 1 November 2009 (UTC)[reply]

Lots of things may have benefited the dinosaurs to our detriment but praise God for the asteroid. Wild or captive anything which might give our molecular adversaries a distinct and overpowering advantage in the long term is still not a good thing even though not present in the short term. Scientists pride themselves on thinking of every possible scenario and that is where the problem lies. Biggerbannana (talk) 04:45, 2 November 2009 (UTC)[reply]

I'm not sure "Scientists pride themselves on thinking of every possible scenario" is really the point. I mean, what? Imagine Reason (talk) 19:15, 2 November 2009 (UTC)[reply]
The problem comes when scientists fail to live up to their expectations and a few unwelcome scenarios slip passed. In such cases we see reactions from others and unfortunately as a matter of pride a complete effort at dismissal. The Titanic is unsinkable. The seals on the Space Shuttle boosters work great. The American Colonies support Britain. I've probably forgotten a few other better examples. Biggerbannana (talk) 21:06, 2 November 2009 (UTC)[reply]
Humans did not coexist with dinosaurs. Please do not use this space to promote a God of some belief system even if you think He willed an asteroid to impact Earth. The Titanic sank because of failed seamanship and inadequate engineering. The seals on the Space Shuttle failed because of failed corporate engineering. The American revolution arose because of failed foreign policy by Britain. No scientists were implicated, least of all any GM biologists who would be relevant to the OP meaning the Original Post not the subsequent debating by the same user. Cuddlyable3 (talk) 22:53, 2 November 2009 (UTC)[reply]

PCR for diagnosis of genital warts

Hi the wikipedia article on genital warts says that less than 1% of people infected with the HPV virus develop visible warts. I'm doing some research into how genital warts are diagnosed in the UK. The only diagnostic tests mentioned on the NHS website are: warts being examined by a doctor and colposcopy. However, if the wikipedia article is right, then 99% of people with genital warts will not be diagnosed by these methods! I've also read that PCR can be used to diagnose genital warts, this could be used to diagnose the vast majority of people who are asymptomatic. My question is does the NHS provide PCR tests for genital warts? Thanks to anyone who can help! RichYPE (talk) 14:26, 1 November 2009 (UTC)[reply]

If you are unable to find this information online, you could try calling the NHS' telephone health information line, NHS Direct. TenOfAllTrades(talk) 15:00, 1 November 2009 (UTC)[reply]
This question verges on the strict "no medical questions" rule, so I'll answer carefully. Useful search terms would be "genito urinary". The NHS wesites I've seen say that they diagnose by looking, and that only visible warts can be treated. Here's some websites: WARNING: REVOLTING IMAGES http://www.chestersexualhealth.co.uk/genitalwarts.htm - and another link that talks about visual diagnosis http://www.nhsdirect.wales.nhs.uk/encyclopaedia/g/article/genitalwarts/. Remember Civility (talk) 15:58, 1 November 2009 (UTC)[reply]
It's a prohibition again medical advice, not medical questions. The OP is clearly not asking for medical advice. --Mr.98 (talk) 16:24, 1 November 2009 (UTC)[reply]
thanks for the clarification. I should have been more precise: "Some may think my answer verges on the strict no medical advice rules, but don't worry, I'm being careful". Is that better? Remember Civility (talk) 19:23, 2 November 2009 (UTC) [reply]
Also deleting a warning against a known hazard as if it were medical advice is extremely careless. Maybe even culpable. Biggerbannana (talk) 18:59, 1 November 2009 (UTC)[reply]
Again, nobody is talking about personal or at-home treatments. (In any case, the treatment plan you mentioned would only be prescribed by a doctor, and would carry copious instructions and warnings and etc. on it anyway.) --Mr.98 (talk) 00:41, 2 November 2009 (UTC)[reply]
My understanding is that acetic acid is not prescribed to the patient by the doctor but is used much as a laboratory stain to reveal altered tissue and therefore no warning is called for between doctor and patient but rather between the pharmaceutical company and the doctor. Common sources of acetic acid (vinegar) do not have warnings in regard to its use as a stain or as a cure for wards by its constant or prolonged application to a wart. Ignoring or thwarting the opportunity to provide such warning here does not accomplish the Wikipedia ban of giving medical advice but rather opens the Wikipedia to a potential charge of culpable negligence. Warnings are not medical advice but the responsibility of anyone with knowledge of the danger to convey that knowledge to others. Biggerbannana (talk) 15:47, 2 November 2009 (UTC)[reply]
Nobody is talking about treatments, and I've never heard of anybody trying to use vinegar at home (I don't really think you know what you are talking about, regarding what doctors prescribe). You are being ridiculous regarding "potential charge of culpable negligence", because Wikipedia is not recommending anyone do anything, much less anything negligent. You might as well argue that Wikipedia is negligent because every time somebody asks about electricity we don't warn people about the risk of electric shock. If you really have a problem with the medical policy here, just take it to the talk page. You are creating an issue here out of nothing and being silly. --Mr.98 (talk) 23:27, 2 November 2009 (UTC)[reply]
I support Mr.98 here. This section is for responses to the OP's question within Ref. Desk policies, not for inventing pseudo-legal accusations. Cuddlyable3 (talk) 11:30, 3 November 2009 (UTC)[reply]

chemical induced orgasam

I've heard that electricity has been used to stimulate the genital region but I do not know if it is to the point of orgasm. Is there chemical which can do this? Biggerbannana (talk) 18:55, 1 November 2009 (UTC)[reply]

SSRI usage can sometimes result in spontaneous orgasms.[20] Red Act (talk) 19:22, 1 November 2009 (UTC)[reply]
Conversely, SSRIs can also prevent orgasm. Mitch Ames (talk) 23:43, 1 November 2009 (UTC)[reply]
Clomipramine, which is a TCA, can also cause spontaneous orgasms. Interestingly, these antidepressant-caused orgasms are sometimes triggered by sneezing.[21] Red Act (talk) 20:12, 1 November 2009 (UTC)[reply]

Effect of optimism on health

I've often heard it claimed as folk wisdom that "positive thinking has been proven to have a positive effect on recovering from illness" or something similar. My understanding is this is likely based off a few hastily done studies in the seventies which probably found nothing more than the obvious link of heavy stress and it wearing out the body. Additionally, further studies that were done extensively failed to show a link. Do we have an article on this, or anything to back it up? I read this in a magazine, and I would like to probe more in depth; my scientific mind finds it very unlikely that the new age baloney about just having a positive attitude vs. a resigned attitude will have any effect on your illness; short of, of course, extreme stress. Magog the Ogre (talk) 20:16, 1 November 2009 (UTC)[reply]

I don't have a lot of knowledge of this topic but I think there is some pretty solid evidence, see PMID 17709956 for example. For your scientific mind, one basic mechanism is that a variety of negative emotions increase the activity of the HPA axis, which leads to increased cortisol secretion, and cortisol has a variety of effects that tend to impair responses to illness, including direct suppression of immune responses. There are currently several whole journals devoted to studying the interactions between the nervous system and the immune system. Looie496 (talk) 21:04, 1 November 2009 (UTC)[reply]
I believe the entire institution of placebos may undermine your pessimistic approach towards optomism. Not to mention psychological disorders, in which the mind is essentially the object of the ailment.Ā :) DRosenbach (Talk | Contribs) 21:06, 1 November 2009 (UTC)[reply]
Actually it was found that people who can find attributions for "why me" or to give meaning for live threatening diseases thay were dealing with are more adaptive and have higher survival rates.--Gilisa (talk) 21:40, 1 November 2009 (UTC)[reply]
To DRosenbach: the placebo has less to do with actually getting better than how one reports his/her symptoms? I.e., did you feel noticeably better when taking this/that drug? To Gilisa: do you have any sourcing? It's precisely this type of thinking for which the author of the magazine article I was reading was critical. Magog the Ogre (talk) 00:02, 2 November 2009 (UTC)[reply]
Did you read the linked article on placebos? The changes are a lot more complicated then simply different reports of symptoms and the article is resonably detailed and sufficiently referenced Nil Einne (talk) 00:32, 2 November 2009 (UTC)[reply]


Look at "The Search for Meaning Following a Storke" by S.Z. Thompson (1991) for a review. It's a bit old one but still invariably cited even in recent articles on similar issues. The effects of negative mood on one's health are obvious and were demonstrated using the most rigorous experimental methods and proven to the molecular level. It's a bit harder to make molecular studies on happiness, but at the least-being happy means not being depressed.--Gilisa (talk) 12:52, 2 November 2009 (UTC)[reply]
I speculate that optimism and good health could both be the result of having sufficient levels of Vitamin D in the body. People in northern latitudes are thought to be deficient during the winter (see Scientific American article from November 2008), and personally I always find the darkening nights of autumn a depressing time, for example. I have seen a scientific paper that speculated that the health benefits of fish could be as a result of their Vitamin D content - do not know if there is any triuth in that. Note that excessive consumption of Vitamin D is harmful and dangerous. 78.146.167.26 (talk) 15:06, 2 November 2009 (UTC)[reply]

How much land is needed to support people?

Hi,

For one average person, or family, in 'the west', lets say Western Europe, how much land in terms of agricultural area is on average needed to support them in terms of food. I know there is no specific amount and the numbers vary widely depending on the habits of countries, eg fish/carbohydrate based diet, and depending on the quality of land in said place but there must be some sort of figure saying something like 'on average one person consumes 1/4 acre of wheat per year' or something like that. Also any figures for other natural resources eg timber, water etc would be useful. Thanks, --86.136.37.191 (talk) 21:47, 1 November 2009 (UTC)[reply]

Someone asked a similar question here. People there gave a wide range of answers, I think it really depends on what kind of crop and how much diversity the people eat. Potatoes are famously compact, while any kind of meat is going to require much more overall. Someone at that link said .5Ha per person for a typical Western diet. I would also think it depends heavily on the quality of the land, specifically whether it gets enough water. Sorry, I guess this turned out to be a pretty waffling responseĀ ;) TastyCakes (talk) 21:55, 1 November 2009 (UTC)[reply]
Oh, also I would imagine if you're in a year round warm climate that gets lots of rain, like much of the equator, each acre is going to be more productive overall and the land needed per capita is going to get smaller. This list gives an idea of this. In Bangladesh, there are apparently 1,946 people per square km of arable land, which is just over .05 hectares per person, and I doubt they are a big net importer of food (although maybe I'm wrong on that). TastyCakes (talk) 22:27, 1 November 2009 (UTC)[reply]
Oh, and this page gives some crop yields in tons per hectare. You have to click on the "Table" icons on the left to see them. TastyCakes (talk) 22:30, 1 November 2009 (UTC)[reply]
The Tables are likely in tonnes per hectare (t/ha) and not tons per hectare.
Yes, sorry that is likely correct. TastyCakes (talk) 04:04, 2 November 2009 (UTC)[reply]

Does phosphorylation occur only with OH / COOH groups?

In order to be phosphorylated, you kind of need to have slight nucleophilicity to begin with right? You can't phosphorylate something if it doesn't have a nucleophilic -OH group to begin with? (The -OH group can be part of a pi system like a carboxylic acid.) John Riemann Soong (talk) 23:08, 1 November 2009 (UTC)[reply]

November 2

Hammerhead Sharks

In what part of the world are hammerhead sharks usually found? The article on them didn't give any specific locations.99.251.239.89 (talk) 00:25, 2 November 2009 (UTC)[reply]

Hammerhead is a genus name and is pretty broad. Click on the individual species links (e.g. Scalloped hammerhead, Great hammerhead) and you'll see little maps. --Mr.98 (talk) 00:45, 2 November 2009 (UTC)[reply]
(edit conflict) That would depend on which shark you wanted. ~ Amory (u ā€¢ t ā€¢ c) 00:48, 2 November 2009 (UTC)[reply]
Cute. DRosenbach (Talk | Contribs) 15:52, 2 November 2009 (UTC)[reply]

h1n1 vaccine

hey is there blood in the h1n1 vaccine? i cant find any info on whats in it.--Least0190 (talk) 00:28, 2 November 2009 (UTC)[reply]

I don't think there is blood (human or otherwise) in any modern vaccine. --Mr.98 (talk) 00:35, 2 November 2009 (UTC)[reply]
However, there are chicken egg proteins in it, so if you have any allergies to chicken eggs, you may not be able to take that vaccine. See also 2009 flu pandemic vaccine for the H1N1 (swine flu) vaccine, and Influenza vaccine for flu vaccines in general. --Jayron32 00:40, 2 November 2009 (UTC)[reply]
(edit conflict) There most definitely is not. The only people who are injected with any sort of blood are heroine addicts. ~ Amory (u ā€¢ t ā€¢ c) 00:40, 2 November 2009 (UTC)[reply]
For proof (sort of) you can read Hemagglutinin (influenza). Essentially, flu surface proteins cause blood to clump in vitro, which would make for a very ineffective vaccine. ~ Amory (u ā€¢ t ā€¢ c) 00:43, 2 November 2009 (UTC)[reply]
{Sidetrack: While I am personally addicted to heroes, I am doubtful about Amory's claim above that heroin addicts are "injected with blood". There may be blood in dirty needles, but there is, as far as I know, no intent to inject blood. ( Blood dopers may inject blood, but that has nothing to do with heroin.) Is there some treatment technique that involves blood injections for addicts?} Bielle (talk) 00:56, 2 November 2009 (UTC)[reply]
Not a treatment by any means - Heroin users who want the hit to hit faster and the high to be higher have been known to stick the needle in their arm, draw a little bit of blood up into the syringe, then inject the mixture back in. It's a great way to OD or get bubbles into your bloodstream, and a really, really, really stupid thing to do, but then so is doing heroin. Trainspotting features this technique, iirc. ~ Amory (u ā€¢ t ā€¢ c) 01:00, 2 November 2009 (UTC)[reply]
I am now better educated. Thank you, Amory. Bielle (talk) 01:04, 2 November 2009 (UTC)[reply]
Honestly this doesn't make sense to me. How would this get the heroin to your brain any faster? I had a vague notion that the idea was precisely to prevent bubbles ā€” once your blood was up in the syringe, you knew there was no air in the needle. I don't know whether that makes sense either, but it's more plausible than that it makes you high faster. --Trovatore (talk) 01:43, 2 November 2009 (UTC)[reply]
Likely there are 2 good reasons for drawing a little blood, 1. you are sure that you are actually in a vein. 2. you ensure that there isn't too much heroin wasted residually in the needle. Unomi (talk) 01:53, 2 November 2009 (UTC)[reply]
Ah right, the vein thing, I think that was it, not the bubbles. --Trovatore (talk) 04:00, 2 November 2009 (UTC)[reply]
Well, for one, some people think that mixing powder with water weakens it, so they just put the powder in the syringe, then use the blood to put it in solution. ~ Amory (u ā€¢ t ā€¢ c) 06:20, 2 November 2009 (UTC)[reply]
Well, there's one pretty blatant error above -- people who get a blood transfusion are certainly injected with blood, obviously. Looie496 (talk) 02:42, 2 November 2009 (UTC)[reply]
I left an obnoxious semantic loophole there for myself - transfusions aren't so much injected, per se, as they are dripped. ~ Amory (u ā€¢ t ā€¢ c) 06:13, 2 November 2009 (UTC)[reply]

ok thanks guys its just that i read that it has pig and horse blood in it, i dont want animals blood in me.--Least0190 (talk) 03:27, 2 November 2009 (UTC)[reply]

If you want it, here's a link. You can read through the full contents of each vaccine, and see that none of them contain anything like that. The closest you get, as Jayron said, is the possibility of less than one millionth of a gram of egg proteins. ~ Amory (u ā€¢ t ā€¢ c) 06:20, 2 November 2009 (UTC)[reply]
May I suggest for you Carl Sagan's The Demon-Haunted World? Imagine Reason (talk) 19:13, 2 November 2009 (UTC)[reply]

Is it possible for energy/ information to travel faster than speed of light in the following case

Let we have a straight pipe AB long but greater than 300,000 km in the free space or vacuum. The pipe is full of small balls of perfect shapes from point A to B and each ball is connected to each other just like two circles at their common tangent point. Assume there is no friction between the pipe and balls. For simplicity we can also ignore the pipe.

Would the last ball at point B be move out if we add a ball to point A of the pipe in less than a second, if not why?

I know nothing can travel greater than speed of light but I'm just asking if it might possible. Thanks 68.147.38.24 (talk) 01:34, 2 November 2009 (UTC) khattak[reply]

Nothing can be perfectly rigid. The balls are made up of atoms. When you push on the first ball, what happens? Its atoms get pushed closer together. They don't want to be closer together, so they start pushing on other atoms behind them, which move farther away. This generates a compression wave down the pipe, which moves quite fast, but slower than light. (It probably moves at about the speed of sound in whatever the balls are made of.) --Trovatore (talk) 01:38, 2 November 2009 (UTC)[reply]

Thank you for your swift reply but sorry to ask you again. Would your answer be the same if two persons commune each other by pushing the same whole pipe or rod in dark space against one another? Because there is diference between the pushing speed of atoms and the pushing speed of the whole pipe/ rod 68.147.38.24 (talk) 03:42, 2 November 2009 (UTC)khattak[reply]

"The whole pipe/rod" is an object composed of atoms. It is no more solid and no more rigid than its full packing of balls. DMacks (talk) 03:48, 2 November 2009 (UTC)[reply]
Trovatore and DMacks are correct. See "Faster-than-light#FTL_phenomena" for some interesting findings. Axl Ā¤ [Talk] 07:10, 2 November 2009 (UTC)[reply]

Supposing the rod was made of quark matter?80.2.195.180 (talk) 12:30, 2 November 2009 (UTC)Trevor Loughlin[reply]

Ho hum, I asked this question a while back - revisiting the archive, I see I forgot to respond, so here's a belated thanks to everyone who answered me. Here is that discussion. Vimescarrot (talk) 13:38, 2 November 2009 (UTC)[reply]
This question is asked so often that it has made me wonder if we have a template for the constant answer that keeps getting rehashed over and over. -- kainawā„¢ 15:09, 2 November 2009 (UTC)[reply]
Hmmm... I just thought that if I have time today, I will trek through the archives and get a list of links to the previous questions. There should be at least 100 of them. Then I can make a template that says: "Your question has been asked and answered here and here and here and here and here and here and here...." -- kainawā„¢ 15:10, 2 November 2009 (UTC)[reply]
Hmm... seems like more work than just answering it again, in most cases (or, putting it another wayā€”it is probably easier for four people to recapitulate parts of an answer than it is for one person to look up all of the previous answers. Also more fun, for the four people). This one is common enough in general that we could have just provided Google linksā€”[22]. --Mr.98 (talk) 16:10, 2 November 2009 (UTC)[reply]
That information cannot travel faster than the speed of light (locally, at least) is a fundamental principle of special and general relativity. Those theories have been very successful at making predictions we have experimentally verified, so we assume that principle is correct. That means that there is a fundamental limit on the rigidity of any object, whatever its composition, since it the speed of sound in it (which is what we are talking about, really) must be slower than the speed of light. --Tango (talk) 18:34, 2 November 2009 (UTC)[reply]
Short answer: in order to change what is happening, you need to apply a force. Either you force something directly, or apply a force to something which applies a force to something else ... We only have four forces - electromagnetism, gravity, strong, and weak, and all four are transmitted (over long distances) at the speed of light. No force travels faster than light, so your effect can't be transmitted faster than light. -- 128.104.112.149 (talk) 22:15, 2 November 2009 (UTC)[reply]
What about Quantum entanglement? From the dictionary: a quantum mechanical phenomenon in which the quantum states of two or more objects have at all times to be described with reference to each other, each instantaneously tracking changes to the other, however large the spatial separation of the objects. Instantaneously, no matter what the distance. That's faster than the speed of light. 20.137.18.50 (talk) 17:51, 3 November 2009 (UTC)[reply]
It's only in a very limited way that quantum entanglement can violate locality. Regardless of quantum entanglement, information can't travel faster than the speed of light, which applies in the case of pushing a long rod. See no-communication theorem. Red Act (talk) 18:42, 3 November 2009 (UTC)[reply]
In short - the quantum states being transmitted are random, so you can't use them to communicate anything other than random values. I did see a proposal that they could be used to communicate a random key for encryption. The encrypted information would then be transmitted by conventional means. --Tango (talk) 18:47, 3 November 2009 (UTC)[reply]
I got rid of the wrong definition at Wiktionary. The original entry, created on August 10, 2005, was okay, if not very edifying. Five days later it was replaced with the incorrect one, which survived for more than four years. Sigh. -- BenRG (talk) 19:49, 3 November 2009 (UTC)[reply]

hox and homeotic transformation

"I have a question/challenge on the homeotic gene page. Are HOX actually homeotic? They definitely are in their ability to regulate A/P development (with anteriorization if a paralogous cluster is knocked out), but there are not homeotic transformations of the limbs in knockouts (HOX 10 and HOX 11 Genes are Required to Globally Pattern the Mammalian Skeleton, Capecchi, et al., 2003, Science Vol 301, p363) From this article, while there are gross limb defects, there is not the reassignment of stylopod to zeugopod, or anything along those lines, making me think that these genes are not completely homeotic in nature. Thoughts? Thanks for maintaining this extremely important page." ā€”Preceding unsigned comment added by Terragamo (talk ā€¢ contribs) 05:06, 2 November 2009 (UTC)[reply]

I believe the HOX genes are unusual in that respect due to the nature of their distinction in limb development. The differential expression of the HOX genes is established during the limb bud stage, when the future limb is at only a small fraction of its final length, barely extended from the imaginal disc. It'll be a day before I can get back to my old developmental textbook, though, so I should note that that's simply my idea. Someguy1221 (talk) 05:12, 2 November 2009 (UTC)[reply]

carbon monoxide

Plants metabolize carbon dioxide. What about carbon monoxide? 71.100.8.110 (talk) 06:20, 2 November 2009 (UTC)[reply]

They're capable of metabolizing carbon monoxide, but the experiments that showed this exposed the plants to carbon dioxide free atmosphere, so I'm not sure how physiologically significant the effect is. One paper describing this: [23] Someguy1221 (talk) 06:26, 2 November 2009 (UTC)[reply]

What happens if a non-depressed person takes anti-depressants?

Not a request for medical advice as I'm just curious about what would happen. Assuming they took them regularly for some time. Would they feel happier, drunk, manic, lose their rational judgment, what? It seems like a scenario for a dystopian science-fiction novel. 78.146.167.26 (talk) 14:58, 2 November 2009 (UTC)[reply]

The answer is going to depend on which specific anit-depressant drug is taken. Googlemeister (talk) 15:39, 2 November 2009 (UTC)[reply]
And the dose. --Mr.98 (talk) 16:13, 2 November 2009 (UTC)[reply]
Don't those drugs sometimes have nasty side-effects? RJFJR (talk) 15:47, 2 November 2009 (UTC)[reply]
Drug abuse and antidepressant might be a good starting point... TastyCakes (talk) 16:25, 2 November 2009 (UTC)[reply]
Terrible withdrawals as well if you forget a dose. Readro (talk) 16:56, 2 November 2009 (UTC)[reply]
Not really. They take several days to begin to take effect. Withdrawal happens after a week or so.--Drknkn (talk) 17:30, 2 November 2009 (UTC)[reply]
That really depends on the drug (and its half-life). Effexor can cause withdrawal with a single missed dose, the same day you missed it. -- Aeluwas (talk) 18:38, 2 November 2009 (UTC)[reply]
I was speaking from personal experience, and you managed to guess my brand of pills in one! CongratulationsĀ :) Readro (talk) 20:53, 2 November 2009 (UTC)[reply]
I didn't know about that drug. I've used Paxil, Prozac, and Zoloft. They work slowly.--Drknkn (talk) 21:15, 2 November 2009 (UTC)[reply]
The vast majority of real-world side effects are mental. Lethargy (laziness), and apathy are common, especially with Zoloft. Anti-depressants do more to curb caring in general than depression. So, you don't feel over-joyed or anything. You'd need a stimulant (like, say, ritalin) for that. If you're already happy, and you take a stimulant, then you'd feel ecstatic. But if you're anxious and you take a stimulant, then the results could be catastrophic.--Drknkn (talk) 17:29, 2 November 2009 (UTC)[reply]
[citation needed]. Comet Tuttle (talk) 18:04, 2 November 2009 (UTC)[reply]
This article suggests that some non-depressed people have become habitual users of anti-depressants for "lifestyle" reasons, perhaps after first being prescribed them for reasons other than depression. The user cited in the article does indeed claim that his daily SSRI makes him feel happier, although it is made clear that the effects of using these drugs in this way are unknown and potentially dangerous. Karenjc 18:40, 2 November 2009 (UTC)[reply]
I don't think it's that simple. Depressants like alcohol, for unknown reasons, make people feel good, while stimulants often dampen their enthusiasm. Imagine Reason (talk) 19:10, 2 November 2009 (UTC)[reply]
Right. I guess it depends on your personality. I've used Paxil, Prozac, and Zoloft over the years. They all make me lethargic and apathetic. Alcohol makes me depressed, but other people get happy when they're drunk, so YMMV.--Drknkn (talk) 21:15, 2 November 2009 (UTC)[reply]
There are different types of anti-depressants. "Tri-cyclics", "MAOI", "SSRI", and then a bunch which are a bit like SSRIs but have different acronyms, such as NASAs, etc. Ann and Bob might have very different experiences even of the same dose of the same medication. There's something called "seretonin syndrome" which can affect people who overdose on soem types of anti-depressant meds. One way to answer the question would be to look at the side-effects of the medications, and the frequency of those side effects, to see what might happen. Don't forget the important Placebo Effect either. Remember Civility (talk) 19:33, 2 November 2009 (UTC)[reply]
Some antidepressants are routinely prescribed for their side-effects, not their anti-depressant effect. See Amitryptiline. I've just come off this drug after 15 years on it for nerve pain: apparently it is the treatment of choice for such pain. I didn't notice a particular anti-depressant effect while I was on it. --TammyMoet (talk) 20:33, 2 November 2009 (UTC)[reply]

the form of energy as matter

Harmonic analysis/synthesis reveals that a combination of sinusoidal frequencies can generate an approximate square wave of any amplitude with a virtually infinite wavelength or zero frequency (assuming that in the case of the expression of energy that the reason absolute zero can not be obtained is the same reason absolute zero frequency can not be obtained). Is it possible that this state of combined frequencies is the state of energy we call energy or matter in the form of a particle? Biggerbannana (talk) 15:25, 2 November 2009 (UTC)[reply]

I'm not really sure what you mean. What are you measuring the frequency of exactly?
The wave function of a particle has a non-zero frequency. See matter wave. Rckrone (talk) 17:23, 2 November 2009 (UTC)[reply]
The idea I am referring to is that of matter or energy being created in a star say by fusion when two atoms of hydrogen becomes one atom of helium. While hydrogen and helium are both matter the energy difference is not expressed in the form of matter but in the form of electromagnetic radiation. The amount of this electromagnetic radiation is known. However, the radiation contains both light and heat and x-rays and Gamma rays and more. However, if you only measure the visible light energy then you come up short and the same for all of the other wavelengths as well but not if you add them all together. When you do add them all together and plot a graph you should get approximately a square wave which itself may have a wavelength near one half the half life of Helium. What I am asking is has anyone else explored this idea and if so have they written a paper? Biggerbannana (talk) 20:52, 2 November 2009 (UTC)[reply]
Animation of the additive synthesis of a square wave with an increasing number of harmonics
A square wave is the sum of an infinite series of sinusoidal waves. Conversely a square wave can be generated by adding together the same series of sinusoidal waves. The frequency of the square wave is that of the lowest frequency sinusoidal wave, so this is not a way to generate a lower frequency wave than one already has. The amplitude of the square wave is less than the amplitude of the lowest frequency sinusoidal wave, shown by the animation. Cuddlyable3 (talk) 22:25, 2 November 2009 (UTC)[reply]
What would say is the lowest electromagnetic frequency produced by the fusion reaction and its amplitude? For that matter how about the amplitude of each frequency produced? 71.100.8.110 (talk) 02:12, 3 November 2009 (UTC)[reply]
A single fusion reaction doesn't create a wide spectrum of EM radiation. The energy released is in the form of just one or a few high energy photons and some kinetic energy. Rckrone (talk) 00:09, 3 November 2009 (UTC)[reply]
Yes and the amplitude and frequency of each...? Have any been measured and added? Biggerbannana (talk) 02:27, 3 November 2009 (UTC)[reply]
They certainly have been measured, but it depends on what reaction it is and on your reference frame. The point I was making is that the Fourier series of a square wave has countably infinite non-zero terms. You can't build it out of 1 or 3 or even 100 frequencies. Rckrone (talk) 06:09, 3 November 2009 (UTC)[reply]
I disagree. Although not a perfect square wave only a few frequencies are necessary to produce a reasonable one with less that maximum amplitude but sufficient duration to trigger a programmable voltage sensor for instance. Biggerbannana (talk) 13:26, 3 November 2009 (UTC)[reply]
But they don't make anything that looks like that. If you have a reaction that sends off one gamma ray, there's no way to argue that's something like a square wave, and the frequency is somewhere on the order of 1019 Hz, which is a bit off from the theorized half life of a proton which would be less than 10-41 Hz. Rckrone (talk) 17:51, 3 November 2009 (UTC)[reply]
Also not sure how you would measure the amplitudes of each of these components. The Fourier coefficients have to go to zero as the frequency go to infinity, but the energy of a photon grows. Rckrone (talk) 06:45, 3 November 2009 (UTC)[reply]

BTW -thanks for whomever provided the graph... 71.100.8.110 (talk) 02:14, 3 November 2009 (UTC)[reply]

YW. User Kief on Commons made the animation and I posted it. Cuddlyable3 (talk) 11:16, 3 November 2009 (UTC)[reply]
Much thanks or hugs and kisses if you are female.Ā ;) Biggerbannana (talk) 13:28, 3 November 2009 (UTC)[reply]

Definition of species

The most common definition of species that I've seen is that a species is a group of organisms that can breed with one another to produce fertile offspring. How are species defined for bacteria and other organisms that reproduce asexually? ----J4\/4 <talk> 18:15, 2 November 2009 (UTC)[reply]

Species#Definitions of species discusses this issue. Basically, the answer is that there is no good answer. --Tango (talk) 18:30, 2 November 2009 (UTC)[reply]
It's a controversial area. See "Species" and "Species problem". There are many species that have been renamed/reclassified: Helicobacter pylori (previously Campylobacter/campylobacter-like organism), Moraxella catarrhalis (previously Branhamella), Pneumocystis jiroveci (previously Pneumocystis carinii), etc.. Axl Ā¤ [Talk] 18:35, 2 November 2009 (UTC)[reply]

Centrifugal force in Earth's orbit

Are there any observable effects of the centrifugal force experienced on the Earth as a result of its orbit around the sun? (Not the centrifugal force resulting from the spinning of the earth on its axis.) For instance, the weight of an object should be slightly greater when it is between the earth and the sun (i.e. daytime) versus when the earth is between it and the sun (i.e. nighttime). I assume that this weight discrepancy would be too small to measure, but is there any such effect that can be measured? Or perhaps I'm thinking wrong about the whole thing? Thanks- Staecker (talk) 19:56, 2 November 2009 (UTC)[reply]

There is a tidal force on the Earth due to the sun that causes a noticeable effect. The tidal force on the Earth due to the moon is about twice as big, though. The tidal forces on the Earth from the moon and to a lesser extent the sun are what causes tides. Red Act (talk) 20:18, 2 November 2009 (UTC)[reply]
Indeed. That tidal force is what the OP is describing - the centrifugal force is equal to the gravitational force (in an inertial frame of reference the gravitational force is a centripetal force and there is no centrifugal force, but if the Earth's frame there is). The difference between the gravitational force between two points on the Earth is called a tidal force. --Tango (talk) 20:27, 2 November 2009 (UTC)[reply]
I'd just add that sometimes the Sun tide and Moon tide reinforce one another (the so-called spring tide), and sometimes partially cancel out (neap tide). This effect has presumably been observed for as long as Man has gone to sea, so yes, the effect is very observable, even if it took a while to figure out the reason. --Trovatore (talk) 20:29, 2 November 2009 (UTC)[reply]
  • Tides have nothing to do with centrifugal force. Tides are due to gravity gradient, i.e. the fact that gravity diminishes with distance. Centrifugal force is due to being in a rotating frame of reference, i.e. in orbit. If the Earth was falling directly into the Sun instead of orbiting, there would by no centrifugal force due to its orbit, but there would still be a tide. Conversely, if the Earth was in a circular orbit but gravity was constant regardless of distance instead of diminishing as 1/rĀ², there would be centrifugal force but no tides. --Anonymous, 06:50 UTC, November 3, 2009.
No, that's not so. The centrifugal force is larger on the midnight side than on the noon side. That still gives you tides. --Trovatore (talk) 06:53, 3 November 2009 (UTC)[reply]
The Effective potential is a way to rewrite the mechanical equations of motion to make it appear that earth's orbit is due to an "effective" force (centrifugal force) - but all that is really happening is that energy and momentum are being conserved. You can interpret the orbit as a "measurable effect" of centrifugal force in this treatment - but you should be aware of the assumptions made by such a formulation. Our effective potential article has a good overview of this topic. Nimur (talk) 20:33, 2 November 2009 (UTC)[reply]
The centrifugal force you're thinking of doesn't exist here. If the Earth were being swung around on the end of a rope, you would feel yourself pressed toward the Earth if you were on the inner side and pushed away from it if you were on the outer side, and that difference would show up on a scale. But the Sun's gravity accelerates you along with the Earth, so there's no such effect. There is a much smaller effect due to the change of the gravitational field with distance (the tidal force already mentioned), but it's not the same thing. For one thing it's symmetricalā€”on the near side you're pulled away from the Earth because you're closer to the Sun, on the far side you're pulled away from the Earth because it's closer to the Sun. For another thing the tidal force drops off with distance from the Sun, while a centrifugal force would increase with distance. -- BenRG (talk) 20:46, 2 November 2009 (UTC)[reply]
Oh please, not this silly quibble again. http://xkcd.com/123 . Oops, sorry, you were making a different point; I hadn't read carefully enough. --Trovatore (talk) 20:48, 2 November 2009 (UTC)[reply]
The weight will be greatest when the body is at right angles to the sun or moon, ie at dawn or dusk for the sun. Dmcq (talk) 20:49, 2 November 2009 (UTC)[reply]
I think BenRG has the right response. I'm pretty sure that what I'm trying to talk about is different from the tidal force, which I know and understand perfectly well. My alleged force would have different properties than the tidal force (read my original description), but if I understand BenRG correctly this effect is cancelled by the sun's gravity. Staecker (talk) 20:54, 2 November 2009 (UTC)[reply]
One way to think of it is that the Sun's gravity cancels the centrifugal force at the Earth's center. On the noon side of the Earth, the Sun's gravity is greater (because you're closer to the Sun) but the centrifugal force is smaller (because the angular velocity is the same, but the radius is smaller), so you're pulled towards the Sun. On the midnight side, the Sun's gravity is diminished, but the centrifugal force is larger, and pulls you out. --Trovatore (talk) 21:05, 2 November 2009 (UTC)[reply]
... of course this ignores the Earth's own rotation. The angular velocity on the noon side is not exactly the same as the angular velocity on the midnight side, because the Earth is spinning. I haven't worked out how much difference this makes. --Trovatore (talk) 21:44, 2 November 2009 (UTC)[reply]

Mass spec

If I wanted to conduct an analysis of a suite of polycyclic aromatic hydrocarbons in a solvent extract of soil using mass spectrometry in order to obtain information on the molecular masses of the constituents what sort of ionisation and insertion techniques should be used? Thanks 188.221.55.165 (talk) 20:01, 2 November 2009 (UTC)[reply]

I'm no analytic chemist, but it sounds like you're talking about GC-MS. Seems that's the way people are measuring soil contaminants these days (refs here: [24],[25],[26] will probably have more detail than WP:RD could provide). --- Medical geneticist (talk) 20:32, 2 November 2009 (UTC)[reply]

Banging your fists from a different perspective

The act of banging your fists on a table, a relatively common gesture, is usually displayed when someone wants to give their speech a firm and absolute meaning, or when that person is angry. What purpose does this gesture serve from an evolutionary perspective, that is, it obviously serves a purpose nowadays to humans, but is this gesture seen in other primates, and how does this gesture in particular serve us for a certain purpose, instead of using a different gesture. Any thoughts? ā€”Preceding unsigned comment added by 201.21.180.57 (talk) 20:43, 2 November 2009 (UTC)[reply]

As a layman, I'd suggest that banging one's fist on the table is no more "innate" than rolling one's eyes; our article Gesture is woefully short but has some interesting-looking references. Comet Tuttle (talk) 20:47, 2 November 2009 (UTC)[reply]
Why make the assumption that this gesture must have an evolutionary "purpose" different than any other gesture? Fist thumping is a useful, easily interpreted gesture. Various primates (including humans) pound their fists on their chests... probably something having to do with trying to establish dominance or display aggression, etc. --- Medical geneticist (talk) 21:53, 2 November 2009 (UTC)[reply]
I'm not sure how biologically relevant it is (I can't find a WP article), but stereotypically male gorillas are thought to pound their chests with their fists in a gesture of physical dominance. Pounding the table could be a related gesture used to connote power and authority. -- 128.104.112.149 (talk) 22:06, 2 November 2009 (UTC)[reply]
Pounding with clenched fist an object such as a table is expressing an urge to Violence that is in conflict with one's unwillingness to hurt a person. Cuddlyable3 (talk) 22:10, 2 November 2009 (UTC)[reply]
[citation needed]. Comet Tuttle (talk) 22:31, 2 November 2009 (UTC)[reply]

Can the Yarkovsky effect be used for space propulsion?

Say you build a probe that heats only part of its exterior using an internal radioactive source. Would you be able to get a net thrust out of this system because of the Yarkovsky effect? I looked around the net for things about such a mechanism but didn't really find anything. 189.15.218.71 (talk) 23:29, 2 November 2009 (UTC)[reply]

Heating only part of the probe would be hard. You would be better off heating all of it and painting different sides different colours so they emit differently (white on one side so it doesn't emit much, black on the other so it does). That kind of thing has been proposed for giving asteroids a slight nudge over a long period of time so they don't crash into the Earth. I've never seen it proposed for spacecraft propulsion. I guess it would work, but it would be really slow - there are better ways. --Tango (talk) 23:42, 2 November 2009 (UTC)[reply]
The problem is that radiation pressure at any reasonable temperature (i.e., a temperature low enough to not melt the propulsion system) is very small, generally measured in micropascals. So plenty of energy may be available from the radioactive source, but you're limited as to how fast you can use that energy, i.e., how much power the propulsion system can provide. That's why nuclear electric rocket designs instead use some kind of electric propulsion system such as an ion thruster, that uses some kind of reaction mass. Red Act (talk) 00:48, 3 November 2009 (UTC)[reply]


November 3

Mil vs Micron

Hi guys, Mil and Microns are measurements used to represent the thickness of plastic. Does any body knows how many microns are equal to 0.55 Mil? ā€”Preceding unsigned comment added by Ferchyn (talk ā€¢ contribs) 01:14, 3 November 2009 (UTC)[reply]

I would guess "Mil" is short for "millimetre" (one thousandth of a metre). A micron is an alternative name for a micrometre (one millionth of a metre). So there are 1000 microns in a mill. That means 0.55 Mil is 550 microns. And I would guess wrong - it's short for milli-inch. So the actual answer is 0.55 Mil is 13.97 microns. --Tango (talk) 01:25, 3 November 2009 (UTC)[reply]
(edit conflict) I was going to correct Tango, but now I don't need to. Our article is at Thou (length). Deor (talk) 01:33, 3 November 2009 (UTC)[reply]
Hmm, in my experience mil is much more common. Probably the article should be moved. --Trovatore (talk) 03:07, 3 November 2009 (UTC)[reply]
I agree, I've never even heard of thou. I'm sure mil is much more common. Red Act (talk) 03:13, 3 November 2009 (UTC)[reply]
Thou is more common in my experience. Perhaps it is a British vs American English thing (me being British)? --Tango (talk) 03:19, 3 November 2009 (UTC)[reply]
e/c Try this Mil to Micron conversion calculator. hydnjo (talk) 01:32, 3 November 2009 (UTC)[reply]
Worth noting is that Google supports unit conversions in their search field: Check it out. TenOfAllTrades(talk) 03:03, 3 November 2009 (UTC)[reply]
If thou is a British thing, then why don't we have the article at Mil (length), since the British use the metric system? ā€”Akrabbimtalk 04:28, 3 November 2009 (UTC)[reply]
Should be, "if thou art a British thing". HTH. --Trovatore (talk) 04:30, 3 November 2009 (UTC) [reply]
Brits use a mixture of metric and imperial. If I need to gap a spark plug I know it's 25 thou, and I always used to gap distributor points to 15 thou. I wouldn't know (although could calculate or look up) the metric equivalent. We mostly inflate tyres to PSI and we always drive miles at MPH. Not very metric, really. --Phil Holmes (talk) 10:47, 3 November 2009 (UTC)[reply]
Oh, I didn't know that. Here in America, all the sciencey people that like metric spin the US as the last barbaric nation to still hold on to feet, pounds, and gallons. ā€”Akrabbimtalk 12:10, 3 November 2009 (UTC)[reply]
No, there are 2 barbaric nations leftĀ ;-) Fribbler (talk) 13:19, 3 November 2009 (UTC)[reply]
Quite recently I was remarking to my wife how I have to mix units to understand the sizes of things without lots of consciuos thought. I only really know my weight in stones and pounds (which, I know, won't help in the US). I run a weather site and follow rain in mm and temperature in Celsius. I'm equally at home with metres or feet or yards. I can only think of fuel consumption in miles per gallon. I used to work on Integrated circuits and couldn't conceive of specifying them in anything other that microns - oh - except their diameter, which is inches. Such is lifeĀ :-) --Phil Holmes (talk) 17:21, 3 November 2009 (UTC)[reply]
Hmm, you did do your silicon work a while ago, eh? The micron is still used (in the sense that no one says micrometer) but it's kind of too big to be a very useful unit. Sometimes comes up when talking about regions of a chip and stuff like that. --Trovatore (talk) 19:11, 3 November 2009 (UTC)[reply]

Best time to buy new houseplants?

I'm a beginning apartment gardener who's ready to move beyond philodendron and spiderplants. I've purchased and read a number of guides and feel ready to buy some new species specimens. I'd like to know if there are guidelines as to the best time to purchase new plants? None of my books have mentioned this, other than notes about protecting plants from cold/heat damage during transit from your nursery to your home. For example, I was wondering if buying plants in spring, when they're waking up and preparing for new growth, makes them more able to adapt to the climate difference in their new location? I'd rather not buy a nice selection of new plants now only to find that they've been too weak to successfully acclimate to their new home... I have an underfloor heated apartment in a climate roughly equivalent to Ohio. Thank you! 218.25.32.210 (talk) 01:29, 3 November 2009 (UTC)[reply]

Your indoor plants are going to have survive dry air conditions. It probably does not matter much when you buy them, but you may get a better price in the autumn when nurseries try to get rid of plants before frost kills them. Graeme Bartlett (talk) 08:29, 3 November 2009 (UTC)[reply]
You'll get a better answer if you just tell us the species you want to grow indoors. One thing you might consider is setting up a heated propagation area (using a heat mat, plastic hoods or vivarium-like enclosures) and instead of buying plants, just grow some from seed, choose the best one as a mother plant, and clone the rest with cuttings. You can do this with several different species, and this will allow you to have some redundancy; If one plant dies, you'll still have the others to work with. Viriditas (talk) 11:05, 3 November 2009 (UTC)[reply]
I actually find it better to buy the plants that are in flower when they are in flower, so I can see whether I like them or not! In the UK, houseplants in garden centres are generally on sale in heated greenhouses or similar environments, so they shouldn't need to acclimatise. --TammyMoet (talk) 11:30, 3 November 2009 (UTC)[reply]

Elements formed by a hydrogen bomb detonation

I've looked on many articles relating to nuclear processes but I haven't found anything particularly straight-forward on the elements created by an atomic explosion, especially hydrogen bombs, which produce high atomic mass elements. Could someone help find/add these elements and add them to an existing article? Much appreciated! Letter 7 it's the best letterĀ :) 01:55, 3 November 2009 (UTC)[reply]

I think the high atomic mass elements will be from the fission part of the bomb (ie. the bit that doesn't involve hydrogen). Nuclear fission product should contain the information you want, but note there aren't specific elements produced - there will be a mixture. --Tango (talk) 02:01, 3 November 2009 (UTC)[reply]
Much obliged! In my rush I didn't see that page, thanks again! Letter 7 it's the best letterĀ :) 02:09, 3 November 2009 (UTC)[reply]
Note that there are some elements that are formed by nuclear reactions other than fission in a bomb. See, e.g., Einsteinium, which is formed by the capture of 15 neutrons by U-238 (which is the sort of thing that you'll only find in a very high neutron economy, of courseā€”like the inside of a bomb, or a particle accelerator). --Mr.98 (talk) 04:41, 3 November 2009 (UTC)[reply]

composite spectral waveform of each element

Have the composite waveforms (harmonic synthesis) of the frequencies and amplitude of the spectral lines for each element been published? Biggerbannana (talk) 05:41, 3 November 2009 (UTC)[reply]

What spectral line are you referring toĀ ? If you mean the emission spectrum, you can find such data at (say) this NIST website. Abecedare (talk) 05:59, 3 November 2009 (UTC)[reply]
Yes, this page seems to point to the spectral data but without graphs and in particular the waveform of the composite emission spectrum for each element. Biggerbannana (talk) 06:18, 3 November 2009 (UTC)[reply]
The wavelength of the composite waveform is the Least common multiple of its component wavelengths and likely to be extremely long i.e. of low frequency. Cuddlyable3 (talk) 11:05, 3 November 2009 (UTC)[reply]

DIY amatuer water quality testing for real nasties like mercury, et alĀ ?

Is it possible for someone without a mass spectrometer // well-equipped laboratory to test water quality for things beyond pH and dissolved oxygen levels? Are there kits one can buy that would reliably identify the presence of heavy metals and such? Or is the only recourse to send a sample to a lab? 218.25.32.210 (talk) 05:48, 3 November 2009 (UTC)[reply]

I've seen at home test kits for mercury, lead, and nitrates. Also things like water hardness and sodium content, but those have less to do with whether the water is potentially harmful. Dragons flight (talk) 06:02, 3 November 2009 (UTC)[reply]
Yes. Aquarium hobbyists measure a range of variables in the aquarium water. These are either test-strips or fluids you mix with water samples and they change colour depending on the concentration of various chemicals. These are available from websites and pet stores. Checking the website of JBL I find test sets for: Iron, copper, ammonium, phosphate, Nitrite, Nitrate, silicic acid, CO2 concentration and Gh and Kh measures of hardness, plus separate calcium measures and a magnesium + calcium indicator.
I also know that in parts of india, the groundwater is tested for arsenic with similar and supposedly inexpensive equipment. EverGreg (talk) 09:49, 3 November 2009 (UTC)[reply]
Mass spec isn't usually used for testing heavy metals in water. Inductively coupled plasma spectroscopy is more common. Rmhermen (talk) 15:26, 3 November 2009 (UTC)[reply]

aluminum-zinc alloys

Help! I'm trying to find a phase diagram for this system and am getting confused. In the first google images hit I find, I get alpha and alpha prime being in the same phase region?! Help?! John Riemann Soong (talk) 06:16, 3 November 2009 (UTC)[reply]

Information on shoulder/chest area anatomy (Medical Science)

Hi, I am writing a novel and I'm trying to make it as realistic as possible (sometimes to a brutal extent). I've come to a point where I need in depth medical information and my local doctor is indisposed. It's also tough to gleam the information I need from several (incomplete) diagrams. So here we are.

Be forewarned, the information I need is for a particularly descriptive(/brutal/violent) fighting scene.

I need to know if there is a name for the area between the shoulder and neck, that is to say, between the Clavicle and Scapula, as this is the point of entry of the character in question's sword. That would be the first part of my question. The second being: I also need a listing of tendons, muscles and organs that a 50cm sword would puncture/cut, if such an action was possible (Not obstructed by bones, etc). Or a source where I can get this. It may carry more relevance than simply determining whether the heart will be among this list, so I will include that it is on the right side of the individual. If the lungs are among this list (Which I believe it will be), am I correct in assuming the individual will cough or gurgle blood in his final moments?

Thank you in advance (In case this question isn't appropriate on this format, I do apologize. I must admit it was rather unclear to me at the time of this posting.) ā€”Preceding unsigned comment added by RyuGenkai (talk ā€¢ contribs) 09:52, 3 November 2009 (UTC)[reply]

Well, that's a nearly 20 inch sword. You can cut anything within 20 inches of where you stab, by aiming at it. Straight down, on the right, you'd hit the apex of the lung, the lung, the diaphragm and probably reach also have the liver en brochette. On the left, the apex of the lung, the pericardium, the heart, the diaphragm and the stomach. Should you decide to go sideways, you could probably get a lung-heart-lung shishkabob. You could cause bilateral pneumothorax and cause death by suffocation without any coughing or gurgling of blood, or if it make for a more dramatic scene you could have blood coughed everywhere. You could transect the carotid and have blood spurting out of the neck, or for a more subdued and dignified death, transect the aorta within the chest cavity and have the victim bleed to death internally with no mess on the carpet. Which is to say: big murder weapon can cause about anything entering about anywhere. Anyway, you should have a look at apex of the lung, which is the area you're entering, and you may get some idea of the anatomy of the area by looking at sternocleidomastoid muscle, scalene muscles, and File:Musculi_coli_base.svg. - Nunh-huh 10:15, 3 November 2009 (UTC)[reply]

If I recall correctly, in Rome (TV series) Marcus Tullius Cicero was assassinated "execution style" by veteran soldier Titus Pullo (Rome character), who killed him with a downward sword thrust similar to the one you describe, as if it were a standard way of killing. In actual history, this is not documented. Edison (talk) 14:32, 3 November 2009 (UTC)[reply]

Thanks folks. Yes, I did mean straight down, don't know why I omitted that. I did say 50cm, but I'm assuming even with the momentum the character has (He's jumping down from above his victim) he won't bury it up to the hilt. At least not with the (limited) knowledge I have of Physiology. I've always had the opinion that humans are a lot more resilient than books and movies make them out to be. I am liking the pneumothorax idea - it opens up new options. I must admit I also have never heard of a historical case of killing in this way, but then again, my character isn't much for history. Nor is he very experienced in the killing business, shall we say. This is the best method he could come up with when presented with a drop from elevation onto the target. RyuGenkai (talk) 16:14, 3 November 2009 (UTC)[reply]

What sort of author capitalizes common nouns like 'physiology,' 'clavicle' and 'scapula.' DRosenbach (Talk | Contribs) 20:43, 3 November 2009 (UTC)[reply]
The film Torn Curtain is notable for a murder scene "that Hitchcock made specifically to show the audience how difficult it is to kill a man". 81.131.65.113 (talk) 21:38, 3 November 2009 (UTC)[reply]

melanin benefit vs cancer risk

Melanin seems to prevent damage to DNA by keeping free radical generation at a minimum. Its deficiency appears to be associated with genetic abnormalities. Is there a "Goldy Locks" level (not too hot and not too cold) of melanocytes to maximize benefit and minimize risk tht can be achieved through selective breeding? Biggerbannana (talk) 13:18, 3 November 2009 (UTC)[reply]

I assume that would vary by the exposure in question, right? That's why melanin content varies in human populations by latitude. The averages of human skin pigments of historic populations at given latitudes is probably close to an ideal "level" for that given latitude, with evolution having found the sweet spot for that level of exposure. --Mr.98 (talk) 13:41, 3 November 2009 (UTC)[reply]
Actually I mean amount of melanocyte cells rather than melanin assuming more cells produce more melanin in total. Biggerbannana (talk) 13:49, 3 November 2009 (UTC)[reply]
Our articles on melanin and melanocyte don't appear to mention any association with genetic abnormalities. Do you have some source for that could be used to improve the articles? 75.41.110.200 (talk) 15:16, 3 November 2009 (UTC)[reply]
The very article you linked to says "The difference in skin color between fair people and dark people is due not to the number (quantity) of melanocytes in their skin, but to the melanocytes' level of activity (quantity and relative amounts of eumelanin and pheomelanin). This process is under hormonal control, including the MSH and ACTH peptides that are produced from the precursor proopiomelanocortin." I didn't know this for sure before I read the article but expected it would be very likely for there to be a big difference in the regulation outside the number of cells as many/most? human systems have rather complex regulation. Nil Einne (talk) 16:20, 3 November 2009 (UTC)[reply]
Has now been by blocked User:TenOfAllTrades, check WT:RD for more. Nil Einne (talk) 16:26, 3 November 2009 (UTC)[reply]

How is heat transfered from one body to another?

and no not a homework question, I assumed it was by the vibrating or moving particles in the hotter substance banging off the particles in the cooler substance/the air in between and causing them to vibrate or move as well. Is that true? ā€”Preceding unsigned comment added by 92.251.255.16 (talk) 17:04, 3 November 2009 (UTC)[reply]

Start with our heat transfer article. DMacks (talk) 17:07, 3 November 2009 (UTC)[reply]
The OP describes heat conduction. The other two ways of heat transfer are radiation and convection. Cuddlyable3 (talk) 20:05, 3 November 2009 (UTC)[reply]

What species is the mushroom in this photo?

I have taken a set of five photos of some mushroom I found in the coastal forest of Poland in October 2009. I think the images have high value but I do not know the type of mushroom in the photos. I have five images in the total set. Here is one of them File:Unidentified_red_mushroom_in_Poland_in_October_2009.jpg and I can post more if need to help identify it. Jason Quinn (talk) 18:00, 3 November 2009 (UTC)[reply]

Amanita muscaria or fly agaric --Tagishsimon (talk) 18:07, 3 November 2009 (UTC)[reply]
That was the first species I found but I don't think it is correct. The mushrooms in my photos (there are the two you see in this photo and a third one in pictures I haven't posted yet) are flatter and less bell-shaped. Jason Quinn (talk) 18:10, 3 November 2009 (UTC)[reply]
Actually, maybe you are correct. Some of the other pictures of them show them to flatter sometimes. I'll wait for more opinions. Thank you. Jason Quinn (talk) 18:12, 3 November 2009 (UTC)[reply]
If you Google image amanita muscaria there is a surprising number of forms displayed. I can't believe that the labelling is so bad. The other point to note is that the cap changes shape as it ages.It opens from an egg shape and will flatten as it matures, some even curling up at the edge, in the final stages before they decompose. Richard Avery (talk) 18:22, 3 November 2009 (UTC)[reply]

Noon in the tropics

Would the variation of the position of the sun, at noon, in the tropics be noticeable. Presumably at some times of the year it is due north, and at others due south. Would it be noticeable? Stanstaple (talk) 18:26, 3 November 2009 (UTC)[reply]

Yes. My Dad used to live in Kuala Lumpur (latitude 3 degrees north) and his condominium had a swimming pool. The temperature of that pool varied widely at different times of the year because the sun would go one side of the building at one time of year and the other side 6 months later. That means that during dry season the swimming pool spent quite a long time in the shadow of the building and during wet season it was in the sun almost the whole time, so was much warmer during the wet season. --Tango (talk) 18:51, 3 November 2009 (UTC)[reply]
Just as in other places, the position of the sun in the sky at the same time of day has range of about 47Ā° (twice the tilt of the Earth's axis) depending on the time of year. Rckrone (talk) 20:14, 3 November 2009 (UTC)[reply]

ptsd

if you had ptsd how long does it last ā€”Preceding unsigned comment added by 24.98.148.83 (talk) 19:43, 3 November 2009 (UTC)[reply]

You mean Posttraumatic stress disorder? According to Posttraumatic stress disorder#Diagnosis, it has to last more than a month to count as PTSD. I think it can last the rest of someone's life in some cases. --Tango (talk) 19:49, 3 November 2009 (UTC)[reply]

Glitch in thermochemistry calculation

Okay, so I have this little problem:

A 100-gram rod of copper (specific heat 0.385 J/g) at 100ĀŗC is immersed in 50 grams of water (specific heat 4.18 J/g) at 26.5ĀŗC. What will the temperature be when both components achieve thermal equilibrium?

And I have tried to solve it as follows:

  • From the information given, we can calculate that when copper cools by 1ĀŗC, it releases 38.5 joules. On the other side, water needs 209 joules to gain 1ĀŗC.
  • Therefore, we can produce the following equations:
and
where x represents the amount of joules gained/lost by the component, and y represents the temperature of the component.
  • To know where the equations intersect, we make them equivalent:
  • We simplify:
  • Now that we know that each component has gained/lost 2,371 joules, we can use that in the original equation to find out the gain/loss of temperature. Let's do it for copper:
  • Therefore, that means that copper has lost 61.59ĀŗC, i.e. its temperature at equilibrium is 38.41ĀŗC. Nevertheless, my textbook gives a value of 37.9ĀŗC. Did I make a mistake, or is this small deviation due to differences in rounding up/down numbers? Thank you. Leptictidium (mt) 19:52, 3 November 2009 (UTC)[reply]
I think you made an error simplifying - I get x=2389.566667, albeit on the back of an envelope and as yet unchecked.

--Tagishsimon (talk) 20:17, 3 November 2009 (UTC)[reply]

I'm sorry for asking what is probably a very stupid question, but where does the 8046.5 come from? Leptictidium (mt) 20:48, 3 November 2009 (UTC)[reply]
It's 38.5*209 ... just some convenient figure allowing me to get rid of both fractions. --Tagishsimon (talk) 20:51, 3 November 2009 (UTC)[reply]

looking at yellow or bright purple stuff for a long time

From my personal observence, okay I wear yellow goggles at night when I sleep. When I wake up when I take off the yellow goggle I still see white stuff for white. If humans look at yellow or purple stuff for a long time I thought white stuff will stay white, and it will not look blue or lime green. Since when I took off the yellow goggle I didn't see white as blue. Will yellow and purple just look less vivid? --209.129.85.4 (talk) 20:22, 3 November 2009 (UTC)[reply]

The human brain compensates for ambient light (wearing yellow goggles would be the same as having the room illuminated by a yellow light). You know what things ought to be white and your brain works out what the ambient light must be and then compensates for it so you see things the colours they would be under white light. If you were in unfamiliar surroundings and didn't know what colours things should be, then your brain might get it wrong or it may take some time to work it out. --Tango (talk) 20:34, 3 November 2009 (UTC)[reply]
Why do you wear yellow goggles when you sleep? Looie496 (talk) 20:39, 3 November 2009 (UTC)[reply]
Corneal transplant--209.129.85.4 (talk) 20:55, 3 November 2009 (UTC)[reply]

UFO identification

I snapped these beautiful critters on Sunday at the Kirstenbosch National Botanical Garden. Can anyone please help identify them? Also, please advise if these pics are the best of their type and pick the best ones between the duplicates. I'm loathe to upload the full res versions if we already have better pictures in our articles. Also, anyone with a good eye for composition feel free to crop these photos (or even just draw borders on the existing) as you see fit. The full res pics are 8MP so I should be able to throw away plenty of pixels and still have something usable.

Photographic critique would be greatly appreciated. Also what's the best way to go about editing these? They are taken with a Canon EOS 350D in Adobe RGB color space, will The GIMP handle it properly? (The low-res versions have been created with Windows Image Resizer PowerToy, causing some colour "bleeding" which I put down to it not playing nice with Adobe RGB.) Or should I just upload the originals and request our WP:Graphic Lab to do the touching up? Regards. Zunaid 20:37, 3 November 2009 (UTC)[reply]

I don't know the answer, but I put them in a gallery for you so I thought I'd leave a message to let you know. They're beautiful, but I don't see much photography. Vimescarrot (talk) 21:08, 3 November 2009 (UTC)[reply]
Thanks. I've edited some captions. Check out the pics in our bee article for some REALLY superb photography. This is rather second-rate in comparison. Zunaid 21:26, 3 November 2009 (UTC)[reply]

vision acuity over 20/400

Is this possible to have vision over 20/400. Do anybody have like 20/1000 or 20/700. From corneal scar I have had my vision was 20/400--209.129.85.4 (talk) 20:56, 3 November 2009 (UTC)[reply]

This article [27] implies that such Snellen fractions do exist. --Cookatoo.ergo.ZooM (talk) 21:14, 3 November 2009 (UTC)[reply]